You are on page 1of 77

2

INDEX
01-08-2019 ........................................................................................................................................................... 3

02-08-2019 .......................................................................................................................................................... 7

03-08-2019 ........................................................................................................................................................ 10

05-08-2019 ........................................................................................................................................................ 14

06-08-2019 ........................................................................................................................................................ 17

07-08-2019 ....................................................................................................................................................... 20

08-08-2019 ........................................................................................................................................................ 23

09-08-2019 ........................................................................................................................................................26

10-08-2019 .........................................................................................................................................................29

12-08-2019 ......................................................................................................................................................... 32

13-08-2019 ......................................................................................................................................................... 35

14-08-2019 .........................................................................................................................................................38

16-08-2019 ........................................................................................................................................................ 40

17-08-2019 .........................................................................................................................................................44

19-08-2019 ......................................................................................................................................................... 47

20-08-2019 ........................................................................................................................................................49

21-08-2019 ......................................................................................................................................................... 52

22-08-2019 ........................................................................................................................................................ 54

23-08-2019 ........................................................................................................................................................ 57

24-08-2019 ....................................................................................................................................................... 60

26-08-2019 ........................................................................................................................................................62

27-08-2019 .........................................................................................................................................................64

28-08-2019 ........................................................................................................................................................ 67

29-08-2019 ........................................................................................................................................................ 71

30-08-2019 ........................................................................................................................................................ 72

31-08-2019 ......................................................................................................................................................... 75

www.shankariasacademy.com | www.iasparliament.com
3

01-08-2019 a. 1 only
1) “Singapore Convention on Mediation‖ b. 2 only
sometimes seen in the news recently is c. Both 1 and 2
associated with which of the following?
d. Neither 1 nor 2
a. New Delhi International Arbitration Centre
(NDIAC)
b. Permanent Court of International Justice 4) Consider the following statements with
(PCIJ) respect to Atal Community Innovation Centre
(ACIC) Program
c. International Centre for Settlement of
Investment Disputes (ICSID) 1. It aims to encourage the spirit of
innovation through solution-driven
d. UN Convention on International Settlement design thinking to serve society.
Agreements (UNISA)
2. It was launched by NITI Aayog under
the Atal Innovation Mission (AIM).
2) Consider the following statements with 3. It aims at spurring community
respect to International Astronautical innovation in underserved and
Federation (IAF) unserved areas of the country.
1. It is an international non-governmental Which of the statement(s) given above is/are
non-profit organization based in Paris. correct?
2. Indian Space Research Organisation a. 1 only
(ISRO) is one among the members of
the organisation. b. 1 and 3 only
3. Each year IAF organizes the c. 2 and 3 only
International Astronautical Congress d. 1, 2 and 3
(IAC), the biggest annual Space event in
the world.
Which of the statements given above are 5) Consider the following statements with
correct? respect to Delhi Public Library (DPL)
a. 1 and 2 only 1. It was established in 1951 with the
financial and technical assistance from
b. 1 and 3 only the UNESCO.
c. 2 and 3 only 2. Its main objective is to provide a free
d. 1, 2 and 3 public library and information services
to the people across the country.
3. It functions under the administrative
3) Consider the following statements with control of the Ministry of Human
respect to “Ghar Ghar Dastak Ghar Ghar Resources and Development (MHRD).
Pustak Scheme”
Which of the statement(s) given above is/are
1. It aims to provide books to the citizens correct?
of Delhi, especially, the residents of
slums, resettlement colonies and rural a. 1 only
areas. b. 1 and 3 only
2. It was launched by the Ministry of c. 2 and 3 only
Human Resources and Development
(MHRD). d. 1, 2 and 3
Which of the statement(s) given above is/are
correct?

www.shankariasacademy.com | www.iasparliament.com
4

6) How Payments Banks differ from Small c. Both 1 and 2


Finance Banks?
d. Neither 1 nor 2
1. While payment banks are to provide
payment solutions, Small Finance
Banks are to provide basic banking 9) Consider the following statements
services. with Coalition of the Willing on Pollinators
2. Small finance can accept deposits and 1. It was founded at the 13th session of the
lend money to customers, whereas Conference of the Parties to the
payments banks cannot lend. Convention on Biological Diversity
(CBD-13) in 2016.
Select the correct answer using the codes given
below: 2. India is a member to this coalition.
a. 1 only Which of the statement(s) given above is/are
correct?
b. 2 only
a. 1 only
c. Both 1 and 2
b. 2 only
d. Neither 1 nor 2
c. Both 1 and 2
d. Neither 1 nor 2
7) Consider the following statements with
respect to Gyre
1. A gyre is a large system of rotating 10) Union Cabinet has recently approved the
ocean currents. setting up of ISRO Technical Liaison Unit
(ITLU) at?
2. The ocean current movement in the
north-Atlantic gyre, as well as north- a. Ottawa, Canada
Pacific gyre, is clockwise, while the
gyres in the Southern hemisphere are b. Paris, France
anti-clockwise. c. Moscow, Russia
Which of the statement(s) given above is/are d. Jakarta, Indonesia
correct?
a. 1 only
Answers
b. 2 only
1. d
c. Both 1 and 2
 Union Cabinet has recently approved the
d. Neither 1 nor 2 signing of the United Nations Convention
on International Settlement
Agreements (UNISA) resulting from
8) Consider the following statements with mediation.
respect to Gambia
Benefits
1. It is located in the East Africa.
 Signing of the Convention will boost the
2. It is almost entirely surrounded by confidence of the investors and shall
Senegal with the exception of its provide a positive signal to foreign investors
western coastline. about India's commitment to adhere to
Which of the statement(s) given above is/are international practice on Alternative Dispute
correct? Resolution (ADR).
a. 1 only UNISA
b. 2 only

www.shankariasacademy.com | www.iasparliament.com
5

 The UNGA adopted the UNISA on 20th  The Atal Innovation Mission (AIM),
December 2018. flagship initiative of NITI Aayog, has
recently launched the Atal Community
 The UNGA authorized that the Convention will Innovation Centre (ACIC) Program to
open for signature at a signing ceremony to be spur community Innovation in underserved
held on 7thAugust 2019 in Singapore and will and unserved areas of the country.
be known as the "Singapore Convention
on Mediation".  The new initiative aims to encourage the spirit
of innovation in the country‘s specific areas
 The Convention provides a uniform and that lack the required infrastructure and
efficient framework for the enforcement of facilitating innovation ecosystem.
international settlement agreements resulting
from mediation and for allowing parties to  The purpose of ACIC is to enable
invoke such agreements. establishment of socially inclusive innovation
ecosystem as well as to encourage equitable
2. d
distribution of necessary infrastructure for
 International Astronautical stimulating innovation across the country.
Federation (IAF) is an international space
 The new initiative offers opportunity to
advocacy and non-
talented youth and researchers in Tier 2, Tier 3
governmental organisation based in Paris.
cities, Smart cities, Aspirational Districts,
 IAF was established in 1951, to foster the North East, J&K, Rural and Tribal regions of
dialogue between scientists around the world India, unserved and underserved regions of
and support international cooperation in all Tier 1 / Metro cities.
space-related activities.
 The ACIC programme would focus in over 484
 Astronautical Society of India, Indian Space districts located in unserved/underserved
Research Organisation (ISRO), Xovian regions of India.
Research & Technologies Pvt. Ltd are the 5. a
members of IAF from India.
 The Delhi Public Library (DPL) was
 Each year IAF in collaboration with the established in 1951 by the then Ministry of
International Academy of Astronautics (IAA) Education with the financial and technical
and the International Institute of Space Law assistance from the UNESCO.
(IISL) organize the International
Astronautical Congress (IAC).  At present, the DPL is functioning under the
administrative control of the Ministry of
 The IAC is the biggest annual Space event in Culture.
the world.
 DPL started with the objective of providing a
 India has the experience of hosting IAC twice free public library and information services to
earlier, in 1988 at Bangalore and in 2007 at the people of National Capital Territory
Hyderabad. (NCT) of Delhi.
3. a 6. c
 Union Culture Ministry has recently  The Postal Department has recently said that
launched the Mobile Library Buses under it has decided to convert the India Post
the Ghar- Payments Bank (IPPB) into a small finance
GharDastakGharGharPustak Scheme of bank (SFB), enabling it to offer small loans to
the Delhi Public Library. customers.
 The Scheme aims to provide books to 7. c
the citizens of Delhi, especially, the
residents of slums, resettlement colonies and  A gyre is a large system of rotating ocean
rural areas. currents.
4. d  In general, currents in the northern
hemisphere travel in the clockwise direction in

www.shankariasacademy.com | www.iasparliament.com
6

a gyre; while currents in southern hemisphere


travel in the anti-clockwise direction (the only
exception is the current direction in the Indian
Ocean, which changes seasonally).
 The ocean current movement in the north-
Atlantic gyre, as well as north-Pacific
gyre, is clockwise, while the gyres in
the Southern hemisphere are anti-
clockwise.

9. a
 The Coalition of the Willing on Pollinators was
founded at the 13th session of the Conference
of the Parties to the Convention on Biological
Diversity (CBD 13) in 2016.
 Nigeria became the fourth African country to
join the Global Coalition of the Willing on
Pollinators
8. b  The coalition now has 28 signatories including
17 European countries, five from Latin
 It is located in the West Africa. America and the Caribbean and four from
 It is almost entirely surrounded by Senegal Africa.
with the exception of its western coastline. 10. c
 It shares its borders with North Atlantic  The Union Cabinet chaired by Prime Minister
Ocean. Narendra Modi has recently approved the
setting up of ISRO Technical Liaison Unit
(ITLU) at Moscow, Russia.
 This will help ISRO to collaborate with Space
agencies/industries in Russia and
neighbouring countries for mutually synergetic
outcomes.

www.shankariasacademy.com | www.iasparliament.com
7

02-08-2019 d. Andhra Pradesh


1) Consider the following statements with
respect to Controller General of Accounts 4) Consider the following statements with
(CGA) respect to Adarsh Smarak Scheme
1. CGA is a constitutional body established 1. It aims to promote basic tourist
under Article 150 of the Indian facilities in well-known historical sites.
Constitution.
2. It is an innovative scheme launched by
2. CGA is the head of the Indian audit & Ministry of Tourism.
accounts department and chief
Guardian of Public purse. Which of the statement(s) given above is/are
correct?
3. CGA is responsible for establishing and
maintaining a technically sound a. 1 only
Management Accounting System. b. 2 only
Which of the statement(s) given above is/are c. Both 1 and 2
correct?
d. Neither 1 nor 2
a. 3 only
b. 2 and 3 only
5) Consider the following statements with
c. 1, 2 and 3 respect to Collagen
d. None 1. It is the single most abundant fat
substances in the animal kingdom.
2) Consider the following statements with 2. These act as supporting structures and
respect to Arbitration and Conciliation anchor cells to each other and give the
(Amendment) Bill 2019 skin strength and elasticity.
1. The bill provides for a time-bound Which of the statement(s) given above is/are
settlement of disputes as well as correct?
accountability of the arbitrator. a. 1 only
2. It seeks to establish an independent b. 2 only
body called the Arbitration Council of
India (ACI). c. Both 1 and 2
Which of the statement(s) given above d. Neither 1 nor 2
is/are incorrect?
a. 1 only
6) With respect to Protection of Children from
b. 2 only Sexual Offences (Amendment) Bill, 2019,
consider the following statements:
c. Both 1 and 2
1. It seeks to enhance punishment for
d. Neither 1 nor 2 sexual offences against children, with a
provision of death penalty.
3) Thudumbattam is a folk art form prevalent 2. The bill, for the first time seeks to
in which of the following states? penalise persons who use children for
pornographic purposes.
a. Kerala
Which of the statement(s) given above is/are
b. Telangana correct?
c. Tamil Nadu a. 1 only

www.shankariasacademy.com | www.iasparliament.com
8

b. 2 only 10) In which of the following, India is not a


member?
c. Both 1 and 2
1. ASEAN
d. Neither 1 nor 2
2. Mekong Ganga Cooperation
3. East Asia Summit
7) Consider the following statements with
respect to National Flag of India Select the correct answer using the codes given
below:
1. It was adopted in its present form
during the meeting of Constituent a. 1 only
Assembly held in July 1947.
b. 1 and 3 only
2. Citizens of India can hoist the Indian
c. 2 and 3 only
flag over their homes, offices and
factories on any day. d. 1, 2 and 3
Which of the statement(s) given above is/are
correct?
Answers
a. 1 only
1. a
b. 2 only
 Controller General of Accounts (CGA), in the
c. Both 1 and 2 Department of Expenditure, Ministry of
d. Neither 1 nor 2 Finance, is the Principal Accounting Adviser
to Government of India.
 He/She is responsible for establishing and
8) Tiger Hills sometimes seen in the news maintaining a technically sound Management
recently is located in which of the following Accounting System.
states?
 He /She exercises the powers of the President
a. Mizoram under Article 150 of the constitution.
b. Karnataka 2. d
c. Jharkhand
 The Arbitration and Conciliation
d. Jammu and Kashmir (Amendment) Bill, 2019 provides for a
time-bound settlement of disputes as well as
accountability of the arbitrator.
9) Consider the following statements with
 It also seeks to establish an independent body
respect to Guinea
called the Arbitration Council of India
1. It is a landlocked country in West (ACI) for the promotion of arbitration,
Africa. mediation, conciliation and other alternative
dispute redressal mechanisms.
2. It shares its borders with Senegal, Mali
and Kenya.  The Council would be mandated to frame
policies for grading arbitral institutions and
Which of the statement(s) given above is/are
accrediting arbitrators.
correct?
3. c
a. 1 only
b. 2 only  Thudumbu is a percussion
instrument from the Kongu region (that
c. Both 1 and 2 includes Coimbatore, Erode, Tiruppur, Karur,
d. Neither 1 nor 2 Namakkal, and Salem districts.) of Tamil
Nadu.

www.shankariasacademy.com | www.iasparliament.com
9

 Thudumbattam, the folk art form, is  Section 2 of the Flag code accepts the right of
prevalent in villages around Karamadai, a all private citizens to fly the flag on their
small town from Coimbatore, Tamil Nadu. premises.
4. a 8. d
 Adarsh Smaarak in an innovative scheme  Tiger Hills is located in Jammu and
launched by Archaeological Survey of Kashmir (J&K).
India (ASI), Ministry of Culture.
 Tiger Hill and Batalik Sector, which hit the
 It aims to promote basic tourist facilities in headlines in 1999 following occupation of
well-known historical sites. several peaks by Pakistani intruders, will be
open to tourists now.
5. b
9. d
 Collagen is a hard, insoluble, and
fibrous protein that makes up one-third  Guinea is a developing country in the tropical
of the protein in the human body. south-western part of West Africa at the
Atlantic Ocean.
 These act as supporting structures and anchor
cells to each other. They give the skin strength
and elasticity.
 U.S Scientists have successfully built
functional heart parts out of collagen using a
3D bio-printer.
6. a
 Under the POCSO Act, 2012, a person is
guilty of using a child for pornographic
purposes if he uses a child in any form of
media for the purpose of sexual gratification.
 The Act also penalises persons who use
children for pornographic purposes resulting
in sexual assault.
 Now, the POCSO Bill, 2019 defines child
pornography as any visual depiction of
sexually explicit conduct involving a child
including photograph, video, digital or
computer generated image indistinguishable
from an actual child.
 The Bill provides that those who use a child for
pornographic purposes should be punished
with imprisonment up to five years and fine.
 However, in the event of second or subsequent
conviction, the punishment would be up to
seven years and fine.
7. c
 The National Flag of India was adopted in its
present form during the meeting of
Constituent Assembly held on the 22 July
1947, a few days before India's independence
from the British on 15 August, 1947.

www.shankariasacademy.com | www.iasparliament.com
10

10. a 3) Consider the following statements with


respect to Bharat Stage (BS) Norms
 India is not a member of ASEAN
1. These are based on European emission
norms.
03-08-2019 2. BS-VI norms will be implemented in
1) Consider the following statements with India from April 1, 2020.
respect to Unlawful Activities (Prevention) 3. Armoured and Specialised Vehicles of
Amendment Bill, 2019 Armed Forces were exempted from BS-
1. It seeks to amend the Unlawful VI Emission Norms.
Activities (Prevention) Act, 1967. Which of the statements given above are
2. The bill for the first time empowers the correct?
Central government to designate an a. 1 and 2 only
individual as terrorist.
b. 1 and 3 only
3. Under the bill, if the investigation is
conducted by an officer of the National c. 2 and 3 only
Investigation Agency (NIA), he/she only d. 1, 2 and 3
requires sanction from the Director
General of NIA.
Which of the statement(s) given above is/are 4) Consider the following statements with
correct? respect to SANKALP Scheme
a. 1 only 1. It aims to implement the mandate of the
National Skill Development Mission
b. 1 and 2 only (NSDM).
c. 1 and 3 only 2. It is an outcome-oriented project
d. 1, 2 and 3 launched by the Ministry of Skill
Development & Entrepreneurship
(MSDE) and supported by the World
2) Consider the following statements with Bank.
respect to Code on Wages Bill, 2019 3. It focuses on the overall skilling
1. Under the Code, the tripartite ecosystem covering both Central & State
committee comprising representatives agencies.
of Central government, State Which of the statement(s) given above is/are
governments and Labour Commission correct?
would fix a floor wage for workers
throughout the country. a. 1 only

2. No state government can fix the b. 1 and 2 only


minimum wage below the minimum c. 1 and 3 only
wage fixed by tripartite committee.
d. 1, 2 and 3
Which of the statement(s) given above is/are
correct?
a. 1 only 5) Consider the following statements with
respect to Institute of Eminence (IoE) Scheme
b. 2 only
1. IoE status will be granted only to public
c. Both 1 and 2 institutions.
d. Neither 1 nor 2 2. The institutions selected under the
scheme will have greater autonomy

www.shankariasacademy.com | www.iasparliament.com
11

compared to other higher educational Which of the statement(s) given above is/are
institutions. correct?
3. All institutions selected under the a. 1 only
scheme will be granted Rs.1000 crore.
b. 2 only
Which of the statement(s) given above
c. Both 1 and 2
is/are incorrect?
d. Neither 1 nor 2
a. 2 only
b. 1 and 3 only
9) Consider the following statements with
c. 1, 2 and 3
respect to Hyperloop technology sometimes
d. None seen in the news recently
1. It envisages pods or capsules travelling
at high speeds through high-pressure
6) Consider the following statements with
tubes erected on columns or tunnelled
respect to Ramon Magsaysay Award
underground.
1. It is the Asia‘s premier prize and highest
2. The vehicle uses magnetic levitation,
honour.
and is propelled by a proprietary
2. It will be awarded annually. electric propulsion system.
3. Since its creation in 1957, no Indian has 3. Motion of hyperloop does not involve
won this award. contact, so the vehicle will be virtually
noiseless.
Which of the statement(s) given above is/are
correct? Which of the statement(s) given above is/are
correct?
a. 1 only
a. 2 only
b. 1 and 2 only
b. 1 and 2 only
c. 1 and 3 only
c. 2 and 3 only
d. 1, 2 and 3
d. 1, 2 and 3

7) Changpa is a tribal community inhabiting in


which of the following states? 10) Consider the following statements with
respect to Tarballs sometimes seen in the news
a. Odisha recently
b. Nagaland 1. They are dark-coloured, sticky balls of
c. Mizoram oil that form when crude oil floats on
the ocean surface.
d. Jammu and Kashmir
2. They are difficult to break down, and
can therefore travel for hundreds of
8) Consider the following statements with miles in the sea.
respect to Pashmina 3. It does not form naturally and the
1. These are ultra-fine cashmere wool presence of tarballs indicates a severe
made up of wool from the Tibetan oil spill.
Antelope. Which of the statements given above are
2. The shawls made up of pashmina wool correct?
were promoted as an alternative to a. 1 and 2 only
Shahtoosh shawl.
b. 2 and 3 only

www.shankariasacademy.com | www.iasparliament.com
12

c. 1, 2 and 3 will get the salary on last day of the week and
daily wagers should get it on the same day.
d. None
 Under the Code, the tripartite committee
comprising representatives of trade unions,
Answers employers and state governments would
fix a floor wage for workers throughout the
1. d country and no state government can fix
 The Bill seeks to empower the central the minimum wage below it.
government to designate an individual a 3. d
―terrorist‖ if they are found committing,
preparing for, promoting, or involved in an act  Bharat Stage (BS) norms are based
of terror. on European emission norms which, for
example, are referred to in a similar manner
 A similar provision already exists in Part 4 and like ‗Euro 4‘ and ‗Euro 6‘.
6 of the legislation (UAPA 1967) for
organisations that can be designated as a  These emission standards were set by the
―terrorist organisation‖. central government to keep a check on the
pollutant levels emitted by vehicles that use
 The existing UAPA law requires an combustion engines.
investigating officer to take prior permission of
the Director General of Police of a state for  To bring them into force, the Central
conducting raids, and seizing properties that Pollution Control Board sets timelines and
are suspected to be linked to terrorist standards which have to be followed by
activities. automakers.

 The amendment Bill, however, removes this  Armoured and Specialised Vehicles of Armed/
requirement if the investigation is conducted Paramilitary Forces were exempted from BS-
by an officer of the National Investigation VI Emission Norms that will Come into Force
Agency (NIA). in April 2020.

 The investigating officer, under the 4. d


Bill, only requires sanction from the Director  Skills Acquisition and Knowledge
General of NIA. Awareness for Livelihood Promotion
2. b (SANKALP) project aims to implement the
mandate of the National Skill
 Parliament has recently passed the Code on Development Mission (NSDM), which
Wages Bill, 2019. was launched by the Ministry of Skill
 The Bill will subsume four labour laws which Development & Entrepreneurship,
are Minimum Wages Act, Payment of Wages through its core sub-missions.
Act, Payment of Bonus Act and Equal  SANKALP is an outcome oriented
Remuneration Act. project supported by World Bank.
 The Code universalises the provisions of  The project will focus on the overall skilling
minimum wages and timely payment of ecosystem covering both Central (MSDE,
wages to all employees irrespective of the NSDA and NSDC) and State agencies,
sector and wage ceiling. and outcomes will be measured through
 At present, the provisions of both Minimum Disbursement Linked Indicators (DLIs) agreed
Wages Act and Payment of Wages Act apply on between MSDE and the Bank.
workers below a particular wage ceiling 5. b
working in Scheduled Employments only.
 This scheme aims to bring higher educational
 It has been ensured in the bill that employees institutions selected as IoEs in top 500 of
getting monthly salary will get the salary by 7th world ranking in the next 10 years and in top
of next month, those working on weekly basis 100 eventually overtime.

www.shankariasacademy.com | www.iasparliament.com
13

 Each Public Institution selected as IoE will  These goats are generally domesticated and
be provided financial assistance up to Rs. 1000 reared by nomadic communities called
Crores over a period of five years. the Changpa in the Changthang region of
Greater Ladakh.
 There will be no financial assistance to
the private institutions.  Central government has granted Geographical
Indication (GI) tag to Kashmir Pashmina in
 Institutions shall be selected as IoEs by the 2008.
Empowered Expert Committee (EEC),
constituted for this purpose.  Bureau of Indian Standards (BIS) has
recently published an Indian Standard for
6. b
identification, marking and labelling of
 Established in 1957, the Ramon Magsaysay Pashmina products to certify its purity.
Award is Asia‟s premier prize and
 The certification will help curb the
highest honour.
adulteration of Pashmina and also protect the
 It is often regarded as the region‘s equivalent interests of local artisans and nomads who
of the Nobel Prize. are the producers of Pashmina raw material.
 It celebrates the memory and leadership  It will also assure the purity of Pashmina for
example of the third Philippine President customers.
Ramon Magsaysay after whom the award is
 The Pashmina shawl was promoted as
named.
alternative to Shahtoosh shawl that is made
 It is given every year to individuals or from the wool of the endangered Tibetan
organisations in Asia who manifest the same Antelope, also known as Chiru.
selfless service and transformative influence
 This high-altitude dwelling animal is mainly
that ruled the life of the late and beloved
found in China's Tibet region.
Filipino leader.
 Although the Shahtoosh trade was banned in
 Indian journalist Ravish Kumar of NDTV
India, the illegal trade continued for a long
news channel was chosen for 2019 Ramon
time, threatening the existence of the Chiru.
Magsaysay Award.
9. c
7. d
 Hyperloop envisages pods or capsules
 The Changthangi goat grows a thick, warn
travelling at high speeds through low-
undercoat which is the source of Kashmir
pressure tubes erected on columns or
Pashmina wool – the world‘s finest
tunneled underground.
cashmere measuring between 12-15 microns in
fiber thickness.  The system is fully autonomous and sealed, so
no driver-related error is anticipated.
 These goats are generally domesticated and
reared by nomadic communities called  In a sealed environment with almost no air
the Changpa in the Changthang region of resistance, the pods are expected to reach
Greater Ladakh of Jammu and Kashmir. very high speeds.
8. b  It uses magnetic levitation, and is
propelled by a proprietary electric propulsion
 Pashmina (meaning made from wool) is made
system.
from the Changthangi or Pashmina goat.
 Motion will not involve contact, so the vehicle
 The Changthangi goat grows a thick, warn
will be virtually noiseless.
undercoat which is the source of Kashmir
Pashmina wool – the world‘s finest cashmere 10. a
measuring between 12-15 microns in fiber
thickness.  Tarballs are dark-coloured, sticky balls of
oil that form when crude oil floats on the
ocean surface.

www.shankariasacademy.com | www.iasparliament.com
14

 They are formed by weathering of crude oil a. 2 only


in marine environments.
b. 1 and 2 only
 They are transported from the open sea to the c. 1 and 3 only
shores by sea currents and waves.
d. 1, 2 and 3
 The presence of several tarballs indicates an oil
spill. But, Tarballs can also occur
naturally from fissures in the ocean floor, 3) Valkyrie sometimes seen in the news
places where oil just gurgles up. recently is
 They are difficult to break down, and can a. A volcanic mountain in Sicily, Italy
therefore travel for hundreds of miles in
the sea. b. A semi-autonomous robot designed by NASA
c. Tribal god worshipped by the Kuki community
of North-East India
05-08-2019
d. None of the above
1) Consider the following statements with
respect to Quick Reaction Surface-to-Air
Missiles (QRSAM) 4) Consider the following statements with
1. It uses liquid-fuel propellant and has a respect to Spodoptera frugiperda sometimes
range of 25-30 km. seen in the news recently

2. It has been developed by the Defence 1. It is an invasive and polyphagous pest,


Research and Development also called as Pink Ball Worm.
Organisation (DRDO) for the India 2. Its deep roots absorb water, leaving the
Army. ground almost dry.
Which of the statement(s) given above is/are Which of the statement(s) given above is/are
correct? correct?
a. 1 only a. 1 only
b. 2 only b. 2 only
c. Both 1 and 2 c. Both 1 and 2
d. Neither 1 nor 2 d. Neither 1 nor 2

2) Consider the following statements with 5) Meghdoot App was recently launched for?
respect to National Agricultural Higher
Education Project (NAHEP) a. To provide crop specific weather based agro
advisories to farmers in local languages
1. It aims to attract talent and strengthen
higher agricultural education in the b. To promote vernacular literatures and to attract
country. the younger generation to the habit of reading

2. This project will be funded by the World c. To commemorate the capture of Siachen Glacier
Bank and the Indian Government on a in the Kashmir region by the Indian Armed Forces
50:50 basis. d. None of the above
3. National Bank for Agricultural and
Rural Development (NABARD) is the
implementing body monitoring this 6) Area 51 sometimes seen in the news recently
project. is located in?

Which of the statement(s) given above is/are a. Antarctica


correct?

www.shankariasacademy.com | www.iasparliament.com
15

b. Russia d. Neither 1 nor 2


c. Greenland
d. United States of America 10) Consider the following pairs
1. Rushikulya River – Odisha
7) It is the largest National Park of India and 2. Bhatsa River – Maharashtra
situated at a high altitude. The park is bounded
3. Uihas River – Madhya Pradesh
on the north by the banks of the Indus River,
and includes the catchments of Markha, Which of the pair(s) given above
Sumdah and Rumbak, and parts of the Zanskar is/are incorrectly matched?
Range. Moreover there is a Famous Tibetan
Buddhist monastery within the park which has a. 1 only
400 year old history. b. 3 only
Which one of the following National Park is c. 2 and 3 only
correctly referred in the given above passage?
d. None
a. Great Himalayan National Park
b. Hemis National Park
Answers
c. Khangchendzonga National Park
1. b
d. Namdhapa National Park
 India has recently test-fired a
sophisticated all-weather and all-
8) Consider the following pairs terrain Quick Reaction Surface-to-Air Missile
(QRSAM) from a test range in Odisha.
1. Farsi Island – Persian Gulf
 The state-of-the-art missile has been
2. Idlib - Turkey developed by the Defence Research and
Which of the pair(s) given above is/are Development Organisation (DRDO) for
correctly matched? the India Army.
a. 1 only  The all-weather and all-terrain missile, which
can be mounted on a truck and stored in a
b. 2 only canister, is equipped with electronic counter
c. Both 1 and 2 measures against jamming by aircraft radars.
d. Neither 1 nor 2  It uses solid-fuel propellant and has a
range of 25-30 km.
2. b
9) Consider the following statements with
respect to Strait of Hormuz frequently seen in  It aims to attract talent and strengthen higher
the news recently agricultural education in the country.
1. It is a strait between Gulf of Aden and  It was funded by the World Bank and
Gulf of Oman. the Indian Government on a 50:50 basis.
2. On its north lies Iran and on its south  Indian Council of Agricultural
lies Saudi Arabia. Research (ICAR) is the implementing body.
Which of the statement(s) given above is/are 3. b
correct?
 Valkyrie is a semi-autonomous
a. 1 only robot designed to operate in hostile
b. 2 only environments has been developed by NASA.

c. Both 1 and 2

www.shankariasacademy.com | www.iasparliament.com
16

 The robot is able to use human tools and can  It is officially known as the Nevada Test and
plot its own path safely across difficult terrain Training Range.
to a location picked by its operator.
 Area 51 is part of the Nellis Air Force Base and
 NASA hopes the robot might one day help is used as a training centre for the US Air
build colonies on the Moon or Mars, but it Force.
could also be used on Earth in places which
cannot be reached by humans.  Several Americans believed it was where the
government hid bodies of aliens and
4. d UFOs, some believed it was where the
government held ―meetings‖ with extra-
 Ministry of Agriculture and Farmers’ Welfare
terrestrials, and others speculated it was where
has recently launched a dedicated website to
the government developed ―time travel‖
help farmers fight against the dreaded Fall
technology.
Armyworm (FAW).
7. b
 FAW (Spodoptera frugiperda) is an
invasive and polyphagous (feeding on many 8. a
foods) pest.
 Iran Revolutionary Guards Corps (IRGC) had
 It can attack cereals and forage grasses. seizes the fuel smuggling foreign tanker
near Farsi Island in the Persian Gulf.
 The website will carry tips and useful
information to prevent and control FAW.
 The website will carry information on
pheromone traps and lures to detect FAW at a
very early stage of infestation.
5. a
 The Ministries of Earth Sciences and
Agriculture have launched a mobile
application that will provide location, and crop
and livestock-specific weather-based agro
advisories to farmers in local
languages.
 To begin with, the service would be available
for 150 districts in different parts of the
country.
9. d
 It will be extended to rest of the country in a
phased manner over the next one year.
 It will provide forecast relating to temperature,
rainfall, humidity, and wind speed and
direction, which play critical roles in
agricultural operations and advisories to the
farmers on how to take care of their crops and
livestock.
 It has been developed by experts from the
India Meteorological Department and Indian
Institute of Tropical meteorology and the
Indian Council of Agricultural Research.
6. d
 Area 51 is located in Southern Nevada, US.

www.shankariasacademy.com | www.iasparliament.com
17

10. b Which of the statement(s) given above is/are


correct?
1. Bhatsa River – Maharashtra
a. 3 only
2. Uihas River – Maharashtra
b. 1 and 2 only
3. Rushikulya River – Odisha
c. 1 and 3 only
d. 1, 2 and 3
06-08-2019
1) Which one of the following best describes the
term Cyber Hygiene? 4) Consider the following statements with
respect to Lalit Kala Akademy
a. It refers to a network of devices that has been
infected with malicious software 1. It is the Government‘s apex cultural
body in the field of performing arts in
b. It relates to the practices and precautions users India.
take to keep sensitive data safe from theft
2. It is an autonomous body, which is fully
c. It refers to a cyber crime which has been done funded by the Ministry of Culture.
cautiously without any evidence
Which of the statement(s) given above is/are
d. None of the above correct?
a. 1 only
2) Consider the following statements with b. 2 only
respect to Section 144 of the Code of Criminal
Procedure (CrPC) c. Both 1 and 2
1. The notification for Section 144 will be d. Neither 1 nor 2
issued by the District Magistrate of the
area.
5) Consider the following statements with
2. State government has no power to respect to Motor Vehicles (Amendment) Bill,
extend the implementation of Sec.144 2019
beyond two months.
1. The Bill requires the central
Which of the statement(s) given above is/are government to constitute a Motor
correct? Vehicle Accident Fund, to provide
a. 1 only voluntary insurance cover to all road
users in India.
b. 2 only
2. The Bill provides for a National Road
c. Both 1 and 2 Safety Board, to be created by the
d. Neither 1 nor 2 central government.
Which of the statement(s) given above is/are
correct?
3) Consider the following statements with
respect to Pathamadai mat a. 1 only

1. It is made of a special kind of grass b. 2 only


called Korai. c. Both 1 and 2
2. They are made by Irulas, a tribal d. Neither 1 nor 2
community native to Nilgris.
3. It was granted Geographical Indication
(GI) tag for Tamil Nadu. 6) Consider the following statements with
respect to Australian Hay Fever

www.shankariasacademy.com | www.iasparliament.com
18

1. It is a tick-borne viral haemorrhagic 1. It is a type of biofuel that is naturally


fever endemic to Australia. produced from the decomposition of
organic waste.
2. There is no medicine for this disease
and can cause even up to death. 2. Manure, food scraps and sewage are
examples of organic matter that
Which of the statement(s) given above is/are
produce biogas.
correct?
Which of the statement(s) given above
a. 1 only
is/are incorrect?
b. 2 only
a. 1 only
c. Both 1 and 2
b. 2 only
d. Neither 1 nor 2
c. Both 1 and 2
d. Neither 1 nor 2
7) Match the following:
Articles – States
10) Consider the following pairs
1. 371 A – a. Sikkim
1. Havana – Mexico
2. 371 C – b. Arunachal Pradesh
2. El Paso – Argentina
3. 371 F – c. Nagaland
Which of the pair(s) given above is/are
4. 371 H – d. Manipur correctly matched?
Select the correct answer using the codes given a. 1 only
below:
b. 2 only
a. 1-a; 2-b; 3-c; 4-d
c. Both 1 and 2
b. 1-d; 2-c; 3-b; 4-a
d. Neither 1 nor 2
c. 1-c; 2-d; 3-a; 4-b
d. 1-d; 2-c; 3-b; 4-a
Answers
1. b
8) Consider the following pairs
 Cyber hygiene relates to the practices and
1. Mulshi Dam – Uttar Pradesh precautions users take with the aim of keeping
2. Pavana Dam – Chhattisgarh sensitive data organized, safe, and secure from
theft and outside attacks.
3. Lal Bahadhur Shastri Dam – Karnataka
2. a
Which of thepair(s) given above is/are
correctly matched?  Section 144 of the Code of Criminal Procedure
(CrPC) is issued in urgent cases of security
a. 3 only threat or riot and bars the assembly of five or
b. 1 and 3 only more people in an area where it has been
imposed.
c. 2 and 3 only
 The notification is issued by the District
d. 1, 2 and 3 Magistrate of the area.
 The section also empowers the authorities to
9) Consider the following statements with block internet access.
respect to Bio Gas  No order under this section (144) can remain
in force for more than two months.

www.shankariasacademy.com | www.iasparliament.com
19

 However, if the state government considers it It will be utilised for:


necessary for preventing danger to human life
1. Treatment of persons injured in road accidents
or for preventing a riot, it can extend the
as per the golden hour scheme
impositions under the sections for not more
than six months from the date of issuance of 2. Compensation to representatives of a person
the initial order. who died in a hit and run accident
3. c 3. Compensation to a person grievously hurt in a
hit and run accident
 Pattamaadai mats are silk mats also called
Pattu paai that originated in a small village in 4. Compensation to any other persons as
Thirunalveli district of Tamil Nadu. prescribed by the central government
 The art and craft of weaving and blending The Bill provides for a National Road Safety Board, to
intricate designs of Pattamadai mats are be created by the central government through a
considered unique to this region. notification.

 Made of a special kind of grass called The Board will advise the central and state
"Korai/Gorai"- it is also called is also called governments on all aspects of road safety and traffic
Korai or Gorai paai. management.

 It was made predominantly by the women 6. d


from the Muslim Lebbai community.  Australian Hay Fever, also known as Allergic
 The conventional method of mat making is a rhinitis, is one of the most common allergic
lengthy process of drying, soaking, splitting disorder affecting one in every five people in
and dyeing the grass. Australia and New Zealand.

 Pattamadai paais are made specially for  Despite its given name, allergic rhinitis is
wedding ceremonies. neither caused by hay or result in a fever.

 It is a token for making the occasion  It usually occurs after the nose or eyes of a
memorable and has the bride and the groom's person come in contact with environmental
names as well as the wedding date woven in it. allergens, such as pollens, dust mite, moulds
and animal hair.
 It was granted GI tag.
 Pollens from grasses are also amongst the
4. b most common causes.
 Lalit Kala Akademi is the Government‘s  The allergic conditions can result in a
apex cultural body in the field of Visual significant impact on sleep, concentration,
arts in India. learning and daily function.
 It is an autonomous body, which is fully  Its immediate symptoms include runny nose,
funded by the Ministry of Culture. sneezing, watery eyes, snoring, fatigue,
headache and throat infection.
 The Akademi is an independent organisation
and functions at arm‘s length from the  Australian Hay Fever is commonly treated
Government. through medications that include
antihistamines, decongestants, nasal sprays
 Sangeet Natak Akademi is the
and Bronchodilators.
government‘s apex cultural body in the field of
Performing arts in India.  The disorder most commonly affects people
5. b between 20-40 years of age.

The Bill requires the central government to constitute a 7. c


Motor Vehicle Accident Fund, to  371 - Special provision with respect to the
provide compulsory insurance cover to all road users states of Maharashtra and Gujarat
in India.

www.shankariasacademy.com | www.iasparliament.com
20

 371A - Special provision with respect to the 07-08-2019


state of Nagaland
1) With respect to the Consumer Protection
 371B - Special provision with respect to the Bill, 2019, consider the following statements:
state of Assam
1. The bill proposes setting up of a Central
 371C - Special provision with respect to the Consumer Protection Authority (CCPA)
state of Manipur to promote, protect and enforce the
rights of consumers as a class.
 371D - Special provisions with respect to the
state of Andhra Pradesh or the state of 2. Under the bill, a manufacturer is now
Telangana responsible for compensation for injury
or damage caused by the defective
 371E - Establishment of Central University in product.
Andhra Pradesh
Which of the statement(s) given above is/are
 371F - Special provisions with respect to the correct?
state of Sikkim
a. 1 only
 371G - Special provision with respect to the
state of Mizoram b. 2 only

 371H - Special provision with respect to the c. Both 1 and 2


state of Arunachal Pradesh d. Neither 1 nor 2
 371-I - Special provision with respect to the
state of Goa
2) With respect to Mount Elbrus, consider the
 371J - Special provisions with respect to the following statements:
state of Karnataka
1. It is the highest peak in the European
8. a Continent.
1. Mulshi Dam – Maharashtra 2. It is an inactive volcano located in the
western Caucasus mountain range,
2. Pavana Dam – Maharashtra Russia.
3. Lal Bahadhur Shastri Dam – Karnataka Which of the statement(s) given above is/are
9. d correct?
 Biogas is a type of biofuel that is naturally a. 1 only
produced from the decomposition of organic b. 2 only
waste.
c. Both 1 and 2
 When organic matter, such as food scraps and
animal waste, break down in an anaerobic d. Neither 1 nor 2
environment (an environment absent of
oxygen) they release a blend of gases, primarily
methane and carbon dioxide. 3) How UN Relief and Works Agency (UNRWA)
differ from UN High Commissioner for
 Because this decomposition happens in an Refugees (UNHCR)?
anaerobic environment, the process of
producing biogas is also known as anaerobic 1. UNRWA deals specifically with
digestion. Palestine refugees whereas UNHCR‘s
mandate is to provide international
10. d protection to refugees worldwide.
1. El Paso – United States of America 2. UNHCR is responsible for Palestine
2. Havana – Cuba refugees outside UNRWA's areas of
operation.

www.shankariasacademy.com | www.iasparliament.com
21

Which of the statement(s) given above is/are 8) Consider the following statements with
correct? respect to Organic Field Effect Transistors
(OFET)
a. 1 only
1. These transistors work on the principle
b. 2 only
that charges in vicinity of channels of
c. Both 1 and 2 certain semiconductors can induce a
current in them.
d. Neither 1 nor 2
2. It is similar to that of the Thin-Film
Transistor (TFT).
4) Russia has recently signed a deal for the Which of the statement(s) given above is/are
supply of uranium for Rooppur Nuclear Power correct?
Plant (RNPP). The plant is located in?
a. 1 only
a. India
b. 2 only
b. Nepal
c. Both 1 and 2
c. Myanmar
d. Neither 1 nor 2
d. Bangladesh

9) Consider the following statements with


5) Aqueduct Water Risk Atlas Report was respect to Canine Distemper Virus (CDV)
released recently by which of the following?
1. It can be transmitted from infected
a. UN – Water dogs
b. World Bank 2. Lions, tigers and leopards are
c. World Resources Institute vulnerable
d. World Wildlife Fund for Nature 3. It attacks the respiratory,
gastrointestinal and nervous systems
Which of the above statements is/are correct?
6) Dixon Plan of 1950 is associated with which
of the following? a. 3 only
a. Naga Insurgency b. 1 and 2 only
b. Reconstruction efforts after Second World War c. 2 and 3 only
c. Presidential form of Government in India d. 1, 2 and 3
d. None of the above
10) Consider the following statements
about Papikonda National Park
7) Which of the following describes the
infiltration plan of Pakistan forces to enter 1. It is located in Andhra Pradesh &
disputed Jammu and Kashmir territory in Telangana
1965 and instigate the locals in starting a 2. It is an Important Bird and Biodiversity
rebellion against Indian sovereignty? Area
a. Operation Polo Which of the above statements is/are correct?
b. Operation Felix a. 1 only
c. Operation Reinhard b. 2 only
d. Operation Gibraltar c. Both 1 and 2
d. Neither 1 nor 2

www.shankariasacademy.com | www.iasparliament.com
22

 Its role encompasses assistance, protection


and global advocacy for Palestine refugees.
Answers
 UNHCR‟s mandate is to provide international
1. c protection to refugees worldwide whenever
 The Consumer Protection bill, 2019, among political conditions allow.
other things, proposes setting up of a Central  UNHCR is responsible for Palestine refugees
Consumer Protection Authority outside UNRWA's areas of operation.
(CCPA) to promote, protect and enforce the
rights of consumers as a class.  UNRWA also provides services to refugees
and people displaced by the Arab-Israeli
 The CCPA would make interventions to conflict of 1967 and subsequent hostilities.
prevent consumer detriment arising from
unfair trade practices. 4. d

 The agency can also initiate class action,  Bangladesh has recently signed a deal
including enforcing recall, refund and return of with Russia for the supply of uranium for its
products. 2,400 MW Rooppur Nuclear Power Plant
(RNPP).
 The Bill also envisages simplified dispute
resolution process, has provision for Mediation  The two countries have also signed 'spent
and e-filing of cases. fuel sent back‟ agreement in 2017 for
management of the nuclear waste of RNPP.
 For the first time there will be an exclusive
law dealing with Product Liability. 5. c

 A manufacturer or product service provider or 6. d


product seller will now be responsible to  Owen Dixon, an Australian jurist chosen by
compensate for injury or damage caused by the United Nations to mediate between India
defective product or deficiency in services. and Pakistan on the J&K issue, in his
2. c report of September 1950, suggested a
package, which did not find acceptance from
 Ministry of Defence has recently flagged off India.
an Expedition Team of Himalayan
Mountaineering Institute (Hmi), Darjeeling  The Plan had assigned Ladakh to India and
to Mt. Elbrus, the highest peak in northern areas and Pakistan-Occupied
European Continent. Kashmir to Pakistan, besides splitting Jammu
between the two.
 It is an inactive volcano located in the western
Caucasus mountain range, near the Georgian  It had proposed a plebiscite in the Kashmir
border in Kabardino-Balkaria and Karachay– valley.
Cherkessia, Russia. 7. d
3. c  Operation Gibraltar was the codename
 The UNRWA has been providing health, given to the strategy of Pakistan to infiltrate
education, relief and social services, as well as Jammu and Kashmir, and instigate the locals
emergency humanitarian assistance, across in starting a rebellion against Indian
its five areas of operations in Jordan, sovereignty.
Lebanon, Syria, West Bank, and the Gaza Strip  Pakistan specifically chose this name to draw a
since 1950. parallel to the Arab invasion of Spain that was
 UNRWA deals specifically with Palestine launched from the port of Gibraltar.
refugees in its five areas of operation in 8. c
Jordan, Lebanon, the Syrian Arab Republic,
West Bank and the Gaza Strip. 9. d
10. b

www.shankariasacademy.com | www.iasparliament.com
23

 It is located in East and West Godavari 1. It was instituted by the Indian Space
districts of Andhra Pradesh. Research Organisation (ISRO).
2. It aims to recognize and reward
journalists who have actively
08-08-2019 contributed towards the field of space
1) With respect to The Jammu and Kashmir science, applications, and research.
Reorganisation Bill, 2019, which was passed Which of the statement(s) given above is/are
recently in the Parliament, consider the correct?
following statements:
a. 1 only
1. The Bill reorganises the state of Jammu
and Kashmir into the Union Territory of b. 2 only
Jammu and Kashmir with a legislature c. Both 1 and 2
and the Union Territory of Ladakh
without a legislature. d. Neither 1 nor 2

2. The High Court of Jammu and Kashmir


will be the common High Court for the 4) UNSC Resolution 47 sometimes seen in the
Union Territories of Ladakh, and news recently is associated with which of the
Jammu and Kashmir. following?
Which of the statement(s) given above is/are a. Jammu and Kashmir Dispute
correct?
b. Israel – Palestine Issue
a. 1 only
c. Declaration of a person as Terrorist
b. 2 only
d. None of the above
c. Both 1 and 2
d. Neither 1 nor 2
5) Consider the following statements with
respect to bilateral trade between India and
2) Consider the following statements with Pakistan
respect to United Nations Industrial 1. India‘s top exports to Pakistan are
Development Organization (UNIDO) vegetable fats and oils and minerals
1. It is one of the specialised agencies of whereas Pakistan‘s top exports to India
U.N. are chemicals and textiles.
2. It promotes industrial development for 2. Pakistan has recently revoked the Most
poverty reduction, inclusive Favoured Nation (MFN) status granted
globalization and environmental to India.
sustainability. Which of the statement(s) given above is/are
Which of the statement(s) given above is/are correct?
correct? a. 1 only
a. 1 only b. 2 only
b. 2 only c. Both 1 and 2
c. Both 1 and 2 d. Neither 1 nor 2
d. Neither 1 nor 2

6) Who among the following were the kings of


3) Consider the following statements with the Kashmir Sultanate?
respect to Vikram Sarabhai Journalism 1. Shah Mir
Award

www.shankariasacademy.com | www.iasparliament.com
24

2. Jamshed a. 1 only
3. Sikandar b. 2 only
4. Zain-ul-Abidin c. Both 1 and 2
5. Fateh Khan d. Neither 1 nor 2
Select the correct answer using the codes given
below:
9) Consider the following statements with
a. 2 and 4 only respect to Quit India movement
b. 1, 3 and 4 only 1. Failure of Cripps Mission is one of the
major reasons for launching Quit India
c. 1, 2 4 and 5 only
Movement by Gandhi.
d. 1, 2, 3, 4 and 5
2. Students, workers and peasants were
the backbone of the movement while the
upper classes and the bureaucracy
7) Consider the following statements with remained largely loyal.
respect to Emex australis Steinh sometimes
seen in the news recently Which of the statement(s) given above
is/are incorrect?
1. It is an invasive weed native only to
Australia. a. 1 only
2. It prospers in conditions such as b. 2 only
drought and unseasonal rain.
c. Both 1 and 2
3. Climate change helps increase its
d. Neither 1 nor 2
numbers.
Which of the statement(s) given above is/are
correct? 10) Consider the following statements with
respect to Forest Survey of India (FSI)
a. 3 only
1. It is a premier national organization
b. 1 and 2 only
under the union Ministry of
c. 2 and 3 only Environment and Forests.
d. 1, 2 and 3 2. Its one of the main objective is to
prepare State of Forest Report
biennially.
8) Consider the following statements with Which of the statement(s) given above
respect to The Right to Information is/are incorrect?
(Amendment) Bill, 2019
a. 1 only
1. It removes the term of Chief
Information Commissioner (CIC) and b. 2 only
the Information Commissioners (IC)
c. Both 1 and 2
and states that central government will
notify the term of office. d. Neither 1 nor 2
2. The Bill states that the salaries,
allowances, and other terms and
conditions of service of the central and Answers
state CIC and ICs will be determined by 1. c
the central government.
2. c
Which of the statement(s) given above is/are
correct?

www.shankariasacademy.com | www.iasparliament.com
25

 UN-Industrial Development Organization accounting for about 70 per cent of its trade
(UNIDO) is the specialized agency that with India.
promotes industrial development for poverty
6. d
reduction, inclusive globalization and
environmental sustainability. 7. c
 The mission of UNIDO is to promote and  Scientists have recently found invasive
accelerate inclusive and sustainable industrial weed Emex australis Steinh that can
development (ISID) in Member States. potentially reduce productivity, in wheat
fields at 25 villages of Faridabad and
 UNIDO is perfectly positioned to assist adjoining districts in Haryana.
developing countries and countries with
economy in transition to shift from HFCs to  The weed, of the family Polygonaceae, is
ozone- and climate-friendly alternatives and to native to Australia and South Africa.
improve energy efficiency.
 It is commonly known as ‗doublegee‘, ‗three
 An agreement was signed recently between corner jack‘ or ‗spiny emex‘.
the National Institute of Solar Energy (NISE)
and the UNIDO to initiate a skill development  This weed prospers in conditions such as
programme for different levels of drought and unseasonal rain.
beneficiaries in the solar thermal energy  Climate change helps increase its
sector. numbers, according to researchers.
3. c 8. c
 As part of centenary year celebrations of Dr. TERM OF OFFICE
Vikram Sarabhai, father of Indian space
programme, ISRO institutes two categories  The RTI Act states that the Chief Information
of awards in journalism. Commissioner (CIC) and Information
Commissioners (ICs) (appointed at the central
 ISRO has announced its ―Vikram Sarabhai and state level) will hold office for a term of
Journalism Award in Space Science, five years.
Technology and Research‖ to recognize and
reward journalists who have actively  The Bill removes this provision and states
contributed towards the field of space science, that the central government will notify the
applications, and research. term of office for the CIC and the ICs.

 The nominations are open to all Indians who SALARY AND ALLOWANCES
have a good experience in journalism.  The Act states that the salary of the CIC and
4. a ICs (at the central level) will be equivalent
to the salary paid to the Chief Election
 UNSC Resolution 47 focuses on the complaint Commissioner and Election Commissioners,
of the Government of India concerning the respectively.
dispute over the State of Jammu and
Kashmir that India took to the Security  Similarly, the salary of the CIC and ICs (at the
Council in January 1948. state level) will be equivalent to the salary paid
to the Election Commissioners and the Chief
5. d Secretary to the state government,
 India granted MFN status to Pakistan in 1996 respectively.
and withdraws it in February 2019.  The Bill seeks to amend these provisions to
 Pakistan is not yet granted MFN status to state that the salaries, allowances, and other
India. terms and conditions of service of the central
and state CIC and ICs will be determined by
 India‘s top exports to Pakistan are chemicals the Central government.
and textiles, accounting for almost 70 per cent
9. d
of the bilateral trade. Pakistan‘s top exports to
India are vegetable fats and oils, and minerals, 10. d

www.shankariasacademy.com | www.iasparliament.com
26

 Forest Survey of India (FSI) is a premier c. Both 1 and 2


national organization under the union
d. Neither 1 nor 2
Ministry of Environment and Forests,
responsible for assessment and monitoring of
the forest resources of the country regularly.
3) Consider the following statements with
 FSI succeeded the Pre-investment Survey of respect to National Entrepreneurship Awards
Forest Resources (PISFR), a project initiated (NEA)
in 1965 by Government of India with the 1. It aims to recognise and honor
sponsorship of FAO and UNDP. outstanding young first generation
 Its main objective is to prepare State of Entrepreneurs and their Ecosystem
Forest Report biennially, providing Builders for their contribution in
assessment of latest forest cover in the country entrepreneurship development.
and monitoring changes in these. 2. It was instituted by the Ministry of Skill
Development and Entrepreneurship
(MSDE).
09-08-2019
Which of the statement(s) given above is/are
1) Consider the following statements with correct?
respect to Bharat Ratna Awards
a. 1 only
1. It is not mandatory that Bharat Ratna be
awarded every year. b. 2 only

2. It cannot be awarded posthumously. c. Both 1 and 2

3. The number of annual awards is d. Neither 1 nor 2


restricted to a maximum of three in a
particular year.
4) Consider the following statements with
Which of the statements given above are respect to Women Entrepreneurship Platform
correct? (WEP)
a. 1 and 2 only 1. It aims to promote and support aspiring
b. 1 and 3 only as well as established women
entrepreneurs in India.
c. 1, 2 and 3
2. It is an initiative of Ministry of
d. None Commerce and Industry.
Which of the statement(s) given above is/are
2) Consider the following statements with correct?
respect to World Biofuel Day a. 1 only
1. It is observed every year to create b. 2 only
awareness about the importance of non-
fossil fuels as an alternative to c. Both 1 and 2
conventional fossil fuels d. Neither 1 nor 2
2. The theme of the World Biofuel Day
2019 is ―Production of Biodiesel from
Used Cooking Oil (UCO)‖. 5) Consider the following statements with
respect to Pradhan Mantri Kisan Maan-Dhan
Which of the statement(s) given above Yojana (PM-KMY)
is/are incorrect?
1. It is a voluntary and contributory
a. 1 only pension scheme for the farmers in the
b. 2 only entry age group of 18 to 40 years.

www.shankariasacademy.com | www.iasparliament.com
27

2. Under the scheme, a monthly pension of 8) Which of the following actions can combat
3,000 rupees will be provided to the deflation?
eligible farmers with up to two acres of
1. Increasing money supply
land-holding on attaining the age of
60years. 2. Decreasing cash reserve ratio
Which of the statement(s) given above is/are 3. Increasing statutory liquidity ratio
correct?
Select the correct answer using the code given
a. 1 only below:
b. 2 only a. 1 and 2 only
c. Both 1 and 2 b. 1 only
d. Neither 1 nor 2 c. 2 and 3 only
d. 1, 2 and 3
6) Consider the following pairs
1. Tungareshwar Wildlife Sanctuary – 9) Consider the following statements
Uttar Pradesh
1. This type of rain is independent of relief
2. Kanjirankulam Bird Sanctuary – Kerala or convection
3. Vaduvoor Bird Sanctuary – Tamil Nadu 2. Basically it is due to the convergence of
two different air masses
Which of the pair(s) given above is/are
correctly matched? 3. This type of rain is also called as frontal
rain.
a. 3 only
Which of the following types of rainfall is
b. 1 and 2 only
correctly related to the above descriptions?
c. 1, 2 and 3
a. Convectional rainfall
d. None
b. Orographic rainfall
c. Temperate Cyclonic rainfall
7) Consider the following statements about
d. Tropical Cyclonic rainfall
Xenotransplants:
1. It is the organ transplantation between
different species. 10) One could not navigate fully in most of the
African rivers. It is mainly because of
2. Hearts, lungs or livers can be
transplanted from one species to the a. They form high and heavy waterfalls
other
b. A large number of rivers flow through narrow
3. It is the most common method used for rift valleys
anti-aging in humans.
c. These rivers flow in the hard bedrocks
Which of the statements given above is/are
correct? d. The Rivers are mostly seasonal in Nature

a. 1 and 2 only
b. 2 and 3 only Answers
c. 1 and 3 only 1. b
d. 1, 2 and 3  The President of India has recently presented
Bharat Ratna Awards to Shri Nanaji
Deshmukh (posthumously),

www.shankariasacademy.com | www.iasparliament.com
28

Dr Bhupendra Kumar  As an important step to catalyse a cultural shift


Hazarika (posthumously) and in youth for entrepreneurship, the Ministry
Shri Pranab Mukherjee at an Investiture of Skill Development and
Ceremony held at Rashtrapati Bhavan. Entrepreneurship (MSDE) has instituted
the National Entrepreneurship Awards
 ‗Bharat Ratna‘, the highest civilian Award of
(NEA).
the country, was instituted in the year 1954.
 It aims to recognise and honor outstanding
 Any person without distinction of race,
young first generation Entrepreneurs and
occupation, position or sex is eligible for these
their Ecosystem Builders for their outstanding
awards.
contribution in entrepreneurship
 It is awarded in recognition of exceptional development.
service/performance of the highest order in
 The National Entrepreneurship Awards (NEA
any field of human endeavour.
2019) is the 4th edition in the NEA series.
 The recommendations for Bharat Ratna are
 The awards seek to instill and entrench the
made by the Prime Minister himself to
entrepreneurial attitude among the future
the President.
generations and the Youth of India.
 No formal recommendations for this are 4. a
necessary.
 The Women Entrepreneurship Platform
 The number of annual awards is restricted to (WEP) is a Government of India initiative
a maximum of three in a particular year. by NITI Aayog
 It is also not mandatory that Bharat Ratna  It aims to promote and support aspiring as
be awarded every year. well as established women entrepreneurs in
 On conferment of the award, the recipient India, assist and handhold them in their
receives a Sanad (certificate) signed by the journey from starting to scaling up and
President and a medallion. expanding their ventures.

 The Award does not carry any monetary 5. c


grant.  Under the PM-KMY, a new Central Sector
 In terms of Article 18 (1) of the Constitution, Scheme, a monthly pension of 3,000 rupees
the award cannot be used as a prefix or will be provided to the eligible farmers with up
suffix to the recipient's name. to two acres of land-holding on attaining the
age of 60years.
 However, should an award winner consider it
necessary, he/she may use the following  It is a voluntary and contributory pension
expression in their biodata to indicate that scheme for the farmers in the entry age group
he/she is a recipient of the award: of 18 to 40 years.

 ‘Awarded Bharat Ratna by the President’  The farmers will have to make a monthly
contribution ranging from 55 to 200 rupees
or depending on their age of entry in the Pension
 ‘Recipient of Bharat Ratna Award’ Fund till they attain the age of 60 years.

2. d 6. a

 World Biofuel Day is observed every year on  Tungareshwar Wildlife Sanctuary –


10th of August to create awareness about the Maharashtra
importance of non-fossil fuels as an alternative  Kanjirankulam Bird Sanctuary – Tamil Nadu
to conventional fossil fuels and highlight the
various efforts made by Government in the  Vaduvoor Bird Sanctuary – Tamil Nadu
biofuel sector. 7. a
3. c

www.shankariasacademy.com | www.iasparliament.com
29

 Xenotransplantation is the transplantation of air is lighter and tends to rise over the cold air
living cells, tissues or organs from one species as shown in figure. In ascent, pressure
to another.Such cells, tissues or organs are decreases the air expands and cools,
called xenografts or xenotransplants. It is condensation takes place and light showers
contrasted with allotransplantation (from called cyclonic or frontal rain occur. The
other individual of same species), Syngeneic heavier and colder air masses eventually
transplantation (Grafts transplanted between pushes up the warmer and lighter air and the
two genetically identical individuals of the sky is clear again
same species) and Autotransplantation (from
10. a
one part of the body to another in the same
person) .  Most rivers in Africa are not navigable due to
waterfalls, weeds and being seasonal. Africa
 Xenotransplantation of human tumor cells
need to combine forces to utilise this rivers for
into immunocompromised mice is a research
irrigation, hydro electric power and combat the
technique frequently used in pre-clinical
big problem of transport by overcoming weeds
oncology research. Human
and waterfalls.
xenotransplantation offers a potential
treatment for end-stage organ failure, a
significant health problem in parts of the
industrialized world. It also raises many novel 10-08-2019
medical, legal and ethical issues. 1) Consider the following statements with
 Xenotransplantation is the transplantation of respect to Samagra Shiksha-Jal Suraksha
living cells, tissues or organs from one species Campaign
to another.Such cells, tissues or organs are 1. It aims to promote water conservation
called xenografts or xenotransplants. It is activities for both rural and urban
contrasted with allotransplantation (from households through mobile phones.
other individual of same species), Syngeneic
transplantation (Grafts transplanted between 2. It was launched by the Ministry of Jal
two genetically identical individuals of the Shakti.
same species) and Autotransplantation (from Which of the statement(s) given above is/are
one part of the body to another in the same correct?
person) .
a. 1 only
 Xenotransplantation of human tumor cells
into immunocompromised mice is a research b. 2 only
technique frequently used in pre-clinical c. Both 1 and 2
oncology research. Human
xenotransplantation offers a potential d. Neither 1 nor 2
treatment for end-stage organ failure, a
significant health problem in parts of the
industrialized world. It also raises many novel 2) Shillong Declaration sometimes seen in the
medical, legal and ethical issues. news recently is associated with which of the
following?
8. a
a. e-governance
9. c
b. Artificial Intelligence
 This type of rainfall is independent of relief or
convection. It is purely associated with c. River Sharing Agreement
cyclonic activity whether in the temperate d. None of the above
regions (depressions) or tropical regions
(cyclones). Basically it is due to the
convergence (meeting) of two different air 3) Consider the following statements with
masses with different temperatures and other respect to Women Transforming India (WTI)
physical properties. As cold air is denser, it Awards
tends to remain close to the ground. The warm

www.shankariasacademy.com | www.iasparliament.com
30

1. Since its inception in 2016, it has been Which of the statement(s) given above is/are
recognising and celebrating stories of correct?
exemplary women from across India.
a. 1 only
2. The theme for WTI Awards 2019 is
b. 2 only
―Women and Entrepreneurship‖.
c. Both 1 and 2
3. It is a flagship initiative of NITI Aayog.
d. Neither 1 nor 2
Which of the statement(s) given above is/are
correct?
a. 1 only 7) Consider the following statements with
respect to National e-Governance Conference,
b. 1 and 3 only
2019 (NCeG – 2019)
c. 2 and 3 only
1. It provides a platform for the delegates
d. 1, 2 and 3 from across the country to share best
practices for achieving effective
governance and public service delivery.
4) Consider the following statements
2. It was jointly organized by the Ministry
1. Diarrhoea was one of the biggest killers of Home Affairs in association with
in children and Rotavirus was one of the Ministry of Electronics & Information
most common causes of severe Technology (MeitY).
diarrhoea in children less than 2 years
Which of the statement(s) given above is/are
of age.
correct?
2. Rotavirus diarrhoea can be prevented
a. 1 only
through vaccination.
b. 2 only
Which of the statement(s) given above is/are
correct? c. Both 1 and 2
a. 1 only d. Neither 1 nor 2
b. 2 only
c. Both 1 and 2 8) Pattachitra, a style of painting is one of the
oldest and most popular art forms of which of
d. Neither 1 nor 2
the following states?
a. Odisha
5) Brunhes–Matuyama refers to?
b. Tamil Nadu
a. Magnetic field flip
c. Maharashtra
b. CERN's Large hadron collider
d. Andhra Pradesh
c. Sixth mass extinct event
d. None of the above
9) Consider the following statements with
respect to PM-KUSUM Scheme
6) Consider the following statements with 1. It is a new umbrella scheme aimed at
respect to Gogabeel Lake ensuring remunerative prices to the
farmers for their produce.
1. It is an ox-bow lake located in
Jharkhand. 2. It is implemented by the Ministry of
Agriculture.
2. It is formed from the flow of the rivers
Mahananda and Kankhar in the north Which of the statement(s) given above is/are
and the Ganga in the south and east. correct?

www.shankariasacademy.com | www.iasparliament.com
31

a. 1 only 5. To encourage Students towards judicious use


and minimum wastage of water at home and
b. 2 only
school level
c. Both 1 and 2
2. a
d. Neither 1 nor 2
 The Department of Administrative Reforms &
Public Grievances (DARPG), in association
with Ministry of Electronics & Information
10) Consider the following statements with
Technology (MeitY) and the State Government
respect to Bhitarkanika National Park
of Meghalaya organized the 22nd National
1. It is located in the state of Odisha Conference on e-Governance (NCeG) 2019 on
8-9thAugust, 2019 at Shillong, Meghalaya.
2. It consists of the Brahmani, Baitrani
Delta.  The theme of this Conference was ―Digital
India: Success to Excellence‖.
3. It was a designated Ramsar site.
Which of the statement(s) given above is/are  At the valedictory session held recently,
correct? ‗Shillong Declaration‘ on e-Governance
was adopted after intensive deliberations
a. 1 only during the sessions held over two days.
b. 1 and 2 only 3. d
c. 1 and 3 only  The WTI Awards, since its inception in 2016,
d. 1, 2 and 3 has been recognising and celebrating stories of
exemplary women from across India.
 The theme for WTI Awards 2019 is ―Women
Answers and Entrepreneurship‖, in continuation
with last edition‘s theme.
1. d
 WTI Awards is a flagship initiative of NITI
 The Department of School Education & Aayog.
Literacy, Ministry of Human Resources
and Development (MHRD) has launched 4. c
the ‗Samagra Shiksha-Jal Suraksha‘  Union Health Ministry has recently decided to
drive to promote water conservation activities provide Rotavirus vaccine to every child across
for School Students, so that they can become all States and Union Territories by September,
competent, conscientious and committed 2019.
water citizens of our nation.
 Rotavirus vaccine was introduced in 2016 in a
 The Department has prepared a detailed phased manner, beginning with 4 States
outline to implement this programme in all the initially and later expanded to 7 more States.
schools of the country.
 The vaccine is now available in 28
Five Major Objectives:
States/Union Territories.
1. To educate Students learn about conservation
 It is expected to be available in all 36
of water
States/Union Territories by September 2019.
2. To sensitize Students about the impact of
5. a
scarcity of water
3. To empower Students to learn to protect the  Earth‘s most recent geomagnetic
natural sources of water reversal — a phenomenon where the planet's
overall magnetic field flips — took at least
4. To help every Student to save at least one litre 22,000 years to complete.
of water per day

www.shankariasacademy.com | www.iasparliament.com
32

 This is more than twice than previously  It aims at boosting farmers‘ income by
thought, according to a study by University of allowing them to sell additional power to the
Wisconsin-Madison. grid through solar plants.
 The magnetic field, which protects the Earth 10. d
from potentially dangerous solar radiation, last
flipped some 770,000 years ago and is
named Matuyama-Brunhes after the 12-08-2019
scientists who discovered it, according to the
journal Science Advances. 1) Consider the following statements with
respect to Indian crested porcupine
6. b
1. It is one of the largest diurnal rodent
 Gogabeel, an ox-bow lake in Bihar‘s Katihar species.
district, has been declared as the state‘s first
‗Community Reserve‘. 2. Its adaptability towards changing
environment helps it to survive in close
 Gogabeel is formed from the flow of the proximity to human beings.
rivers Mahananda and Kankhar in the
Which of the statement(s) given above is/are
north and the Ganga in the south and east.
correct?
 It is the fifteenth Protected Area (PA) in Bihar.
a. 1 only
7. a
b. 2 only
 The Department of Administrative c. Both 1 and 2
Reforms & Public
Grievances (DARPG), Ministry of d. Neither 1 nor 2
Personnel, Public Grievances and Pensions,
in association with Ministry of Electronics
& Information Technology (MeitY) and 2) Consider the following statements with
the State Government of Meghalaya organized respect to Oussudu Lake
the 22nd National Conference on e-
1. It is a manmade lake located in
Governance (NCeG) 2019 on 8-9thAugust,
Puducherry and Tamilnadu.
2019 at Shillong, Meghalaya.
2. The structure of the lake consist of
 The theme of this Conference was ―Digital water, wetland/marsh and mudflats.
India: Success to Excellence‖.
Which of the statement(s) given above is/are
 At the valedictory session held recently, correct?
‗Shillong Declaration‘ on e-Governance
was adopted after intensive deliberations a. 1 only
during the sessions held over two days. b. 2 only
8. a c. Both 1 and 2
 Pattachitra style of painting is one of the oldest d. Neither 1 nor 2
and most popular art forms of Odisha.
9. d
3) Consider the following statements with
 Kisan Urja Suraksha evam Utthaan respect to Surrogacy (Regulation) Bill 2019
Mahabhiyan (KUSUM) is a farmer-
oriented solar power scheme that will 1. The Bill prohibits commercial
allow setting up grid-connected solar plants in surrogacy, but allows altruistic
rural areas and off-grid solar pumps. surrogacy.

 It involves decentralized solar power 2. Under the bill, unmarried women is not
production of up to 28,250 Megawatt (Mw) eligible to become a surrogate mother.
over five years.

www.shankariasacademy.com | www.iasparliament.com
33

Which of the statement(s) given above is/are a. 1-2-3-4


correct?
b. 4-3-2-1
a. 1 only
c. 1-3-2-4
b. 2 only
d. 3-1-2-4
c. Both 1 and 2
d. Neither 1 nor 2
7) Consider the following statements with
respect to Repurpose Cooking Oil (RUCO)
Initiative
4) Consider the following statements with
respect to Transgender Persons (Protection of 1. It aims to collect and convert the used
Rights) Bill, 2019 cooking oil into bio-fuel.
1. The Bill defines a transgender person as 2. It was launched by the Food Safety and
one whose gender does not match the Standards Authority of India (FSSAI).
gender assigned at birth.
Which of the statement(s) given above is/are
2. Under the bill, no government or correct?
private entity can discriminate against a
a. 1 only
transgender person in employment
matters, including recruitment, and b. 2 only
promotion.
c. Both 1 and 2
Which of the statement(s) given above
is/are incorrect? d. Neither 1 nor 2

a. 1 only
b. 2 only 8) Consider the following pairs

c. Both 1 and 2 Places in News – Country

d. Neither 1 nor 2 1. Aden – Yemen


2. Temple Mount – Italy

5) Almatti is a major dam built across which of Which of the pair(s) given above is/are
the following rivers? correctly matched?

a. R. Cauvery a. 1 only

b. R. Godavari b. 2 only

c. R. Krishna c. Both 1 and 2

d. R. Tungabadra d. Neither 1 nor 2

6) Arrange the following states in ascending 9) Consider the following statements with
order based on the forest and tree cover (In respect to Jungle cat
terms of %): 1. This animal is quite unusual and is
1. Haryana found in scrub jungles, dense forests,
foot hills and grasslands.
2. Punjab
2. It has been listed under the Least
3. Rajasthan Concern Category in the IUCN Red List.
4. Arunachal Pradesh Which of the statement(s) given above is/are
correct?
Select the correct answer using the codes given
below:

www.shankariasacademy.com | www.iasparliament.com
34

a. 1 only  Lake acts as the single largest catchment of


fresh water in Puducherry.
b. 2 only
c. Both 1 and 2  The vegetation of the lake (ranges from small
herbs to trees) supports migratory avifauna as
d. Neither 1 nor 2 well as native birds during summer and winter.
3. c
10) Consider the following statements with  The Bill prohibits commercial surrogacy, but
respect to Black-naped hare allows altruistic surrogacy.
1. This animal is restricted to India and Sri  Altruistic surrogacy involves no monetary
Lanka. compensation to the surrogate mother other
2. In many places, they are considered than the medical expenses and insurance
pests because of the damage they can do coverage during the pregnancy.
to young trees and to crops.  Commercial surrogacy includes surrogacy or
Which of the statement(s) given above is/are its related procedures undertaken for a
correct? monetary benefit or reward (in cash or kind)
exceeding the basic medical expenses and
a. 1 only insurance coverage.
b. 2 only Eligibility criteria for surrogate mother
c. Both 1 and 2 The surrogate mother has to be:
d. Neither 1 nor 2 1. A close relative of the intending couple
2. A married woman having a child of her own
Answers 3. 25 to 35 years old
1. b 4. A surrogate only once in her lifetime
 One of the largest nocturnal rodent species, 5. Possess a certificate of medical and
the Indian crested porcupine lives in psychological fitness for surrogacy
burrows, caves and rock crevices during the
4. d
day.
 The Bill defines a transgender person as
 The porcupine is known for the many spines
one whose gender does not match the
on its body and when alarmed, it raises its
gender assigned at birth.
spines and rattles those on its tail.
 It includes transmen and trans-women,
 The fact that they are able to adapt to changing
persons with intersex variations, gender-
environmental conditions and food habits
queers, and persons with socio-cultural
helps this species survive in close proximity to
identities, such as kinnar and hijra.
human beings.
2. c  Intersex variations is defined to mean a person
who at birth shows variation in his or her
 The Ousteri Lake also referred to as Oussudu primary sexual characteristics, external
Lake is a man-made lake situated genitalia, chromosomes, or hormones from the
in Puducherry and Tamil Nadu. normative standard of male or female body.
 It is recognized as one of the important  No government or private entity can
wetlands of Asia by the International Union for discriminate against a transgender person in
Conservation of Nature and Natural Resources employment matters, including recruitment,
(IUCN). and promotion. Every establishment is
required to designate a person to be a
 The structure of the lake is complex consisting complaint officer to deal with complaints in
of water, wetland and mudflats. relation to the Act.

www.shankariasacademy.com | www.iasparliament.com
35

5. c Jerusalem that for thousands of years has


been venerated as a holy site, in Judaism,
 Almatti, a major dam on Krishna Christianity, and Islam alike.
River that is located in Karnataka, close to the
Maharashtra border. 9. c
6. a  This animal is quite unusual and is found in
scrub jungles, dense forests, foot hills and
 Assessment of forest and tree cover are done grasslands.
by the Forest Survey of India, whose findings
are published in the India State of Forest  It is an excellent ambush predator that hunts
Report (ISFR). small rodents, ground dwelling birds, lizards
and insects.
 The last report was published in 2017.
 Its population, despite it being in the Least
 A quarter of India‘s geographical area (24.49 Concern category in the IUCN Red List,
%) is under forest and tree cover. is declining.
 Lowest forest and tree cover (In terms 10. c
of %): The lowest forest and tree cover in the
country is in Haryana, at 6.79 % of its  Black naped hare (L. nigricollis), also
geographical area. known as the Indian hare, is restricted
to India and Sri Lanka.
 Punjab follows with 6.87
%. Rajasthan‟s forest and tree cover is over  It is usually found in the scrubs, forests
7.26 % of its geographical area while Madhya adjacent to farm lands and dry rocky habitats,
Pradesh‟s is 27.73 %. feeding on young grass blades and other herbs
depending upon their availability.
 Highest forest and tree cover (In terms
of %): At 97 %, the Union Territory of  These creatures are known to live in burrows.
Lakshadweep is on top – its geographical area
 They come out during the night and early
is only 30 square km.
mornings for grazing.
 Among the six states with the highest forest
 In many places, L. nigricollis are considered
and tree cover, four are in the Northeast –
pests because of the damage they can do to
Manipur, Arunachal Pradesh, Mizoram
young trees and to crops.
and Meghalaya.
 Goa and Kerala are two other states with more
than 50 % of their geographical area under 13-08-2019
forest and tree cover.
1) Which of the following reasons are
 Highest forest and tree cover (In terms responsible for resurgence of infectious
of area): In terms of geographical area, the diseases like diphtheria and pertussis?
highest tree and forest cover is in Madhya
Pradesh, with 85,487 sq km. 1. Low awareness of vaccination

7. c 2. Loss of herd immunity


3. Negligence due to misinformation about
 The Food Safety and Standards
vaccination
Authority of India (FSSAI) has launched
the Repurpose Cooking Oil (RUCO) Select the correct answer using the codes given
initiative to collect and convert used cooking below:
oil into bio-fuel.
a. 2 only
8. a
b. 1 and 2 only
 The Temple Mount, known to Muslims as
c. 2 and 3 only
the Haram esh-Sharif and the Al Aqsa
Compound is a hill located in the Old City of d. 1, 2 and 3

www.shankariasacademy.com | www.iasparliament.com
36

5) The term ‗Perseids‘ often seen in news is


related to
2) Consider the following statements
regarding PRAAPTI portal a. Radio Galaxy
1. It is launched by the Ministry of New b. Crater lake
and Renewable Energy.
c. Meteor shower
2. It is developed to bring transparency in
d. Tropical cyclone
power purchase transaction between
power generators and distribution
companies.
6) Consider the following statements
Which of the above statement(s) regarding Minamata Convention
is/are incorrect?
1. It is a global treaty to protect human
a. 1 only health and environment from the
adverse effects of Mercury.
b. 2 only
2. From 2025, the convention will ban the
c. Both 1 and 2
production, import and export of
d. Neither 1 nor 2 products that contain mercury.
3. Global Environment Facility (GEF)
serves as the financial mechanism of the
3) Consider the following statements with
convention.
respect to Tardigrades
Which of the above statements are correct?
1. They are also called as water bears, or
moss piglets. a. 1 and 2 only
2. They are tiny animals that can survive b. 2 and 3 only
extremes of temperature and pressure
c. 1 and 3 only
including the frigid vacuum of space.
d. 1, 2 and 3
3. They have been found everywhere, from
mountain tops to the deep sea and mud
volcanoes, from tropical rain forests to
the Antarctic. 7) Consider the following statements
regarding Measles
Which of the statement(s) given above is/are
correct? 1. It is a highly contagious bacterial
disease.
a. 1 only
2. Under the Global Vaccine Action Plan,
b. 1 and 3 only measles and rubella are targeted for
elimination by 2020.
c. 2 and 3 only
Which of the above statement(s) is/are
d. 1, 2 and 3
correct?
a. 1 only
4) Kajin Sara Lake sometimes seen in the news
b. 2 only
recently is located in which of the following
countries? c. Both 1 and 2
a. China d. Neither 1 nor 2
b. Japan
c. Nepal 8) Article 130 of the Indian Constitution deals
with which of the following?
d. Srilanka
a. Appellate Jurisdiction

www.shankariasacademy.com | www.iasparliament.com
37

b. Appointment of ad hoc Judges  Recently, Central Electricity Authority has


requested state distribution companies to list
c. Seat of Supreme Court
payment dues in the portal.
d. Original jurisdiction of the Supreme Court
3. d
 Tardigrades are also known as water bears or
9) Consider the following statements with moss piglets.
regards to the Organization of Islamic
Cooperation (OIC)  They are a phylum of water-dwelling eight-
legged segmented micro-animals.
1. It has permanent delegations from both
the United Nations Organisation and the  They have been found everywhere, from
European Union. mountaintops to the deep sea and mud
volcanoes, from tropical rain forests to the
2. Iran is a member of this organization Antarctic.
3. India is not a member of this  They are among the most resilient animals
organization. known, with individual species able to survive
Which of the given above statements are extreme conditions.
correct?  They have survived exposure to outer space.
a. 1 and 2 only
 They can survive extremes of temperature and
b. 1 and 3 only pressure, including the frigid vacuum of space.
c. 2 and 3 only  It has been recently revealed that the
Beresheet mission was carrying a cargo of
d. 1, 2 and 3
dehydrated microscopic lifeforms known as
tardigrades.
10) With reference to the Transfer Pricing, 4. c
which of the following statements is correct?
 The Kajin Sara lake in Manang district of
a. It refers to the method of transferring the Nepal was discovered about a few months ago
subsidy for Farmers by a team of mountaineers.
b. It refers to the rules and regulations of foreign  This newly-discovered lake in Nepal is likely
remittances by the Individuals to set a new record of being the world's highest
c. It refers to the rules and methods for pricing the lake replacing Tilicho, which is situated at an
transactions between enterprises under common altitude of 4,919 metres in the Himalayan
ownership or control nation and currently holding the title.

d. It refers the method of determining the 5. c


corporate taxes by the Government on Domestic 6. c
Companies
 The convention came into force in 2017.
 It will ban the production, import and export
Answers of production that contain mercury
1. d from 2020.

2. a  Recent report has asserted that climate change


and the rising ocean temperatures means fish
 PRAAPTI (Payment Ratification And Analysis are more active and need more food to survive.
in Power procurement for bringing Consuming more prey means consuming more
Transparency in Invoicing of generators) mercury.
portal is launched by the Ministry of Power.

www.shankariasacademy.com | www.iasparliament.com
38

 Union Cabinet approved a proposal for 1. It is a subspecies of the clouded leopard


ratification of the Minamata Convention in that is also found in India.
2018.
2. Habitat destruction and overhunting for
7. d their skins are the major reasons for
their
 It is a highly contagious viral disease. decline.
 Under the plan, it targets elimination of
measles and controlling of Rubella by Which of the statement(s) given above is/are
2020. correct?
8. c a. 1 only
9. d b. 2 only
10. c c. Both 1 and 2
d. Neither 1 nor 2
14-08-2019
1) Injeti Srinivas Committee sometimes seen in 5) With respect to Noctilucent clouds, consider
the news recently is associated with which of the following statements
the following?
1. They are extremely rare very high
a. Insolvency Bankruptcy Code clouds seen in the night sky, usually on
cloudy, winter nights.
b. Hague convention on Child Abduction
2. They occupy troposphere and are only
c. Corporate Social Responsibility Reforms
seen at latitudes between 45°N and 80°N
d. None of the above in the Northern Hemisphere, and
equivalent latitudes in the southern
hemisphere.
2) REGN-EB3 and mAb114 sometimes seen in
Which of the statement(s) given above is/are
the news recently are?
correct?
a. Drugs for Ebola Treatment
a. 1 only
b. Exo planets
b. 2 only
c. Bio-Molecular Scissors
c. Both 1 and 2
d. Asteroids found in Kuiper belt
d. Neither 1 nor 2

3) ULTRASAT sometimes seen in the news


6) Consider the following statements with
recently is a microsatellite to record black
respect to Panchamirtham of Palani Temple
holes to be launched by which of the following
countries? 1. It is a combination of five natural
substances – banana, jaggery, cow ghee,
a. Israel
honey and cardamom.
b. China
2. Not even a single drop of water is added
c. Russia during the preparation of the
panchamirtham.
d. India
3. It has been recently granted the
Geographical Indication (GI) tag.
4) With respect to Formosan Clouded Leopard, Which of the statement(s) given above is/are
consider the following statements correct?

www.shankariasacademy.com | www.iasparliament.com
39

a. 1 only 2. It was established to receive,


consolidate, manage all contributions
b. 1 and 2 only
towards disaster management around
c. 2 and 3 only the world.
d. 1, 2 and 3 Which of the statement(s) given above is/are
correct?
a. 1 only
7) Consider the following statements with
respect to Central Bureau of Investigation b. 2 only
(CBI)
c. Both 1 and 2
1. It is a statutory body established
d. Neither 1 nor 2
through Central Vigilance Commission
Act, 2003.
2. The superintendence of CBI lies solely Answers
with the Central Vigilance Commission
(CVC). 1. c

Which of the statement(s) given above is/are  The High Level Committee on Corporate
correct? Social Responsibility (CSR) headed by
Corporate Affairs Secretary Injeti
a. 1 only Srinivas has recommended making CSR
b. 2 only expenditure tax deductible and that
compliance violations be treated as a civil
c. Both 1 and 2 offence that attracts penalties.
d. Neither 1 nor 2 2. a
 The drugs, named REGN-EB3 and mAb114,
8) Swachh Nagar App which was launched work by attacking the Ebola virus with
recently for which of the following purposes? antibodies, neutralising its impact on human
cells.
a. Waste Collection
 They are the "first drugs that, in a scientifically
b. Grievance Redressal sound study, have clearly shown a significant
c. Sweeper Alert Scheme diminution in mortality" for Ebola patients.
d. None of the above 3. a
 The satellite, known as ULTRASAT, will carry
a telescope designed to observe the Universe as
9) Usha Thorat Committee sometimes seen in never seen before and will operate in the
the news recently is associated with which of ultraviolet range of light that is normally
the following? invisible to humans.
a. Shareholder Norms  Israel plans to launch into space in 2023 an
b. Corporate Governance "innovative" scientific satellite with a unique
space telescope that will investigate and record
c. Offshore Rupee Markets cosmic explosions and black holes.
d. Economic Capital Framework
 The project will be led by Weizmann Institute
of Science (WIS) and the Israel Space Agency
(ISA) at the state's Ministry of Science and
10) Consider the following statements with Technology.
respect to Special Purpose Trust Fund (SPTF)
4. c
1. It is a specific fund housed within the
UN Secretariat. 5. d

www.shankariasacademy.com | www.iasparliament.com
40

 Noctilucent clouds are extremely rare very 10. a


high clouds seen in the night sky, usually
 India has contributed one million US Dollars
on clear, summer nights.
to the United Nations Special Purpose Trust
 Noctilucent clouds are extremely rare Fund for the Resident Coordinator System.
collections of ice crystals, occasionally
 According to the UN information, Special
appearing in late clear summer evenings after
Purpose Trust Fund (SPTF) is a specific fund
sunset, but before it gets completely dark.
housed within the UN Secretariat,
 They become visible about the same time as established to receive, consolidate, manage all
the brightest stars appear and often stay visible contributions and financial transactions of the
after dark because they are still reflecting new Resident Coordinator system in a
sunlight due to their great height. transparent and effective way.
 They are higher up than any other clouds,  SPTF web portal, displays in real time all
occupying the layer of atmosphere known as commitments, contributions and expenditures
the Mesosphere, and are only seen recorded for the fund.
at latitudes between 45°N and 80°N in the
 Funding sources include voluntary
Northern Hemisphere, and equivalent
contributions from the Member States, the
latitudes in the southern hemisphere.
doubling of cost-sharing amounts from the UN
 They are seen less often in the southern entities who are members of the United
hemisphere as there is very little land and very Nations development system and a one per
few people there. cent levy applied to contributions for UN
development related activities earmarked to a
 Only the southern tip of Argentina and Chile, single agency, single programme or project.
and the Antarctic are at the correct latitude.
6. d
7. d
16-08-2019
1) Consider the following statements with
 CBI derives power to investigate from the respect to World Trade Organisation (WTO)
Delhi Special Police Establishment Act, 1946.
1. WTO has defined the criteria for a
 The superintendence of CBI related member country to be classified as
to investigation of offences under developed or developing.
the Prevention of Corruption Act, 1988
lies with the Central Vigilance 2. WTO allows member countries to
Commission (CVC) and in other classify themselves as developed or
matters with the Department of developing or least developed countries.
Personnel & Training (DOPT) in the 3. No WTO member can challenge the
Ministry of Personnel, Pension & Grievances of decision of a member country to be
the Government of India. classified as developing.
8. a Which of the statement(s) given above is/are
 Swachh Nagar app that will allow users to correct?
have waste — solid, wet or construction and a. 2 only
demolition (C&D) — collected by their urban
local bodies was launched recently by the b. 1 and 2 only
Union Housing and Urban Affairs Ministry. c. 1 and 3 only
9. c d. None
 The Reserve Bank of India‘s (RBI) task force,
chaired by Usha Thorat, set up to examine
the issues relating to the offshore rupee 2) Consider the following statements with
markets has submitted its report to the respect to Vir Chakra Award
governor recently.

www.shankariasacademy.com | www.iasparliament.com
41

1. It is the third highest gallantry award 1. It is a panel that is formed for a specific
awarded for acts of bravery in the in the period of time by the house or the
presence of the enemy. speaker of the house.
2. The Medal cannot be awarded 2. Consultative committees are also a part
posthumously. of the parliamentary committees.
3. Officers and men and women of all Which of the statement(s) given above is/are
ranks of the Naval, Military and Air correct?
Forces are eligible for this award.
a. 1 only
Which of the statement(s) given above is/are
b. 2 only
correct?
c. Both 1 and 2
a. 3 only
d. Neither 1 nor 2
b. 1 and 2 only
c. 1 and 3 only
6) Consider the following statements with
d. 1, 2 and 3
respect to Purse seine
1. It is a fishing method in which large
3) Tricalcium Phosphate is commonly used for weighted nets are dragged across the
which of the following purposes? ocean floor.
a. Weedicide 2. It is mainly used to catch fish species
that swim in large schools near the
b. Fire extinguisher
ocean surface.
c. Bone substitution
Which of the statement(s) given above is/are
d. Manufacture of crackers correct?
a. 1 only
4) Consider the following statements with b. 2 only
respect to Foreign Trade Act (FTA), 1992
c. Both 1 and 2
1. Its main objective is to provide a frame
d. Neither 1 nor 2
work for development and regulation of
foreign trade by facilitating imports into
the country.
7) Consider the following statements with
2. According to FTA, Gift items of value up respect to Varadharaja Perumal Temple
to Rs.5,00,000 received from foreign
1. It is a temple dedicated to Lord Shiva.
countries to people residing in India are
exempted from customs duty. 2. It is located in Kancheepuram, Tamil
Nadu.
Which of the statement(s) given above is/are
correct? 3. It is one among the 108 Divya Desams.
a. 1 only Which of the statement(s) given above is/are
correct?
b. 2 only
a. 2 only
c. Both 1 and 2
b. 1 and 2 only
d. Neither 1 nor 2
c. 2 and 3 only
d. 1, 2 and 3
5) Consider the following statements with
respect to Parliamentary Committee

www.shankariasacademy.com | www.iasparliament.com
42

8) Consider the following statements with


respect to SAMRATH Scheme
Answers
1. It is a scheme for capacity building and
skilling in the sunrise sector. 1. d

2. Under the scheme, preference will be  There are no WTO definitions of


given to various social groups, SC, ST, ―developed‖ and ―developing‖ countries.
differently abled, minorities and other  WTO allows member countries to classify
vulnerable groups. themselves as ―developed‖ or ―developing‖.
Which of the statement(s) given above is/are
 However, other members can
correct?
challenge the decision of a country to be
a. 1 only classified as ―developing‖.
b. 2 only  Only the ―least-developed countries‖
status is designated by the UN.
c. Both 1 and 2
d. Neither 1 nor 2  (i.e) WTO recognizes as least-developed
countries (LDCs) those countries which have
been designated as such by the United
Nations.
9) Which of the following countries is/are
recognised by WTO as “Developed Countries”? 2. c
1. China  The Vir Chakra is awarded for acts of
2. Russia gallantry in the presence of the enemy,
whether on land or at sea or in the air.
3. Japan
 The Medal may be awarded posthumously.
4. Australia
Following categories of persons are eligible for the Vir
Select the correct answer using the codes given Chakra:
below:
1. Officers and men and women of all ranks of
a. 2 only the Naval, Military and Air Forces, of any of
b. 3 and 4 only the Reserve Forces, of the Territorial Army,
Militia and of any other lawfully constituted
c. 2 and 4 only Armed Forces.
d. None 2. Matrons, Sisters, Nurses and the Staff of the
Nursing Services and other Services pertaining
to Hospitals and Nursing, and Civilians of
10) Consider the following statements with either sex serving regularly or temporarily
respect to Iceland under the orders, directions or supervision of
any of the above-mentioned Forces.
1. It is a volcanic Island located in the
middle of the North Atlantic Ocean. 3. c
2. Iceland is in many ways reminiscent of  Tricalcium Phosphate is a commonly
the surface of Mars. used bone substitute material from
Which of the statement(s) given above is/are natural sources.
correct? 4. a
a. 1 only  The basic objective of Foreign Trade
b. 2 only Development and Regulation (FTDR)
Act 1992 is to provide a frame work for
c. Both 1 and 2 development and regulation of foreign trade by
d. Neither 1 nor 2 facilitating imports into the country, as well as,

www.shankariasacademy.com | www.iasparliament.com
43

taking measures to increase exports from India  The size of purse seine nets can be varied,
and any other related matters. depending on what species is being targeted.
 According to According to the Foreign Trade  It is not only depriving traditional
Act, gift items of a value of up fishermen of catch but threatening the
to Rs. 5,000 received from foreign countries marine ecosystem as a whole in the region.
to people residing in India are exempted
from customs duty.
 Domestic industry players brought to the
notice of the Customs Department that several
e-commerce companies, especially from China,
were delivering orders to their India-resident
customers by disguising their products as gifts
so as to evade the Customs duty that would
normally be applicable on the products.
5. a
A parliamentary committee means a committee that:
1. Is appointed or elected by the House or
nominated by the Speaker /Chairman.
2. Works under the direction of the Speaker /
Chairman.
3. Presents its report to the House or to the
Speaker / Chairman.
4. Has a secretariat provided by the Lok Sabha /
Rajya Sabha. 7. c
The consultative committees, which also consist of  Varadharaja Perumal Temple is a Hindu
members of Parliament, are not parliamentary temple dedicated to Lord Vishnu located in
committees as they do not fulfill above four conditions. the holy city of Kanchipuram, Tamil
Nadu.
6. b
 It is one of the Divya Desams, the 108 temples
Purse seine fishing of Vishnu believed to have been visited by the
 It is mainly used to catch fish species that 12 poet saints, or Alwars.
swim in large schools near the ocean surface. 8. b
 The main species fished with purse seine gear  The ‗Scheme for Capacity Building in Textile
are southern bluefin tuna, Australian sardine, Sector (SCBTS)‘ shall be known by the name
blue mackerel and jack mackerel. Samarth‖, signifying the broad objective of the
Working scheme to skill the youth for gainful and
sustainable employment in the textile sector.
 In a purse seine the top of the net is floated at
the ocean‘s surface and the bottom of the net  Its objective is to provide demand driven,
has weights attached that pull the walls of the placement oriented National Skills
net downwards. Qualifications Framework (NSQF)
compliant skilling programmes to incentivize
 The bottom of the net has a wire threaded and supplement the efforts of the industry in
through it which is pulled and tightens the net creating jobs in the organized textile and
like a purse trapping the fish inside. related sectors, covering the entire value chain
 The net is then pulled in toward the boat and of textile, excluding Spinning and Weaving.
the catch is either pumped or lifted out with 9. b
small nets or the whole net is brought aboard.

www.shankariasacademy.com | www.iasparliament.com
44

d. Neither 1 nor 2

2) Consider the following statements with


respect to Chief of Defence Staff (CDS)
1. CDS is meant to be a single-point
military advisor to the government on
important defence and strategic issues.
2. It is a high military office that oversees
and coordinates the working of the
three Services.
3. Before the establishment of the post of
CDS, National Security Adviser (NSA)
advises India‘s Prime Minister on
military matters.
Which of the statement(s) given above is/are
correct?
a. 2 only
10. c
b. 1 and 2 only
 Iceland, a volcanic island in the middle of the
c. 2 and 3 only
North Atlantic, is in many ways reminiscent
of Mars. d. 1, 2 and 3
 To prepare for the next mission to Mars in
2020, NASA has taken to the lava fields of
3) Consider the following statements with
Iceland to get its new robotic space explorer
respect to Tirur betel vine
ready for the job.
1. It is mainly cultivated in the state of
 With its black basalt sand, wind-swept dunes Tamil Nadu.
and craggy peaks, the Lambahraun lava field at
the foot of Iceland's second biggest glacier, 2. Its juice is used to heal bone fracture.
Langjokull, was chosen as a stand-in for the 3. It is primarily used for food packaging.
Red Planet's surface.
Which of the statement(s) given above is/are
correct?
17-08-2019 a. 3 only
1) Consider the following statements with b. 2 and 3 only
respect to Tawlhlohpuan
c. 1, 2 and 3
1. It is a medium to heavy, compactly
woven, good quality fabric from the d. None
state of Assam.
2. It has been recently granted GI tag. 4) Consider the following statements
Which of the statement(s) given above is/are 1. It is a shawl/textile from Mizoram.
correct?
2. It is an important marriage outfit in the
a. 1 only state.
b. 2 only 3. It was granted GI tag recently.
c. Both 1 and 2 Identify the product that correctly matches
with the above description:

www.shankariasacademy.com | www.iasparliament.com
45

a. Feni a. 1 only
b. Puanchei b. 2 only
c. Kullu Shawl c. Both 1 and 2
d. Pashmina shawl d. Neither 1 nor 2

5) Consider the following statements with 8) Which of the following features are
respect to Shimla Conference of 1945 envisaged as the bulwarks of the Indian
Constitution?
1. It was held between Viceroy Lord
Linlithgow and major political leaders 1. Election Commission of India
in British India.
2. Comptroller and Auditor-General of
2. The meeting was convened to seek India
approval of Indian leaders for the
3. Union Public Service Commission
Wavell Plan aimed to usher in Indian
self-rule. 4. State Public Service Commission
Which of the statement(s) given above is/are Select the correct answer using the codes given
correct? below.
a. 1 only a. 1 and 2 only
b. 2 only b. 1, 2 and 3 only
c. Both 1 and 2 c. 1, 3 and 4 only
d. Neither 1 nor 2 d. 1, 2, 3 and 4

6) Consider the following pairs 9) The Traditional Drum named as ―KA BOM‖,
is associated with which one of the following
1. Operation Pawan – IPKF In Sri Lanka
North-East Indian tribal community?
2. Operation Rakshak – Jammu and
a. Aptanis
Kashmir
b. Hajong
3. Operation Rhino – Assam
c. Angami
Which of the pair(s) given above is/are
correctly matched? d. Khasis
a. 1 only
b. 1 and 2 only 10) Consider the following pairs
c. 2 and 3 only Places in News – Country
d. 1, 2 and 3 1. Sanaa – Oman
2. Gibraltar – Overseas territory of Spain
7) Consider the following pairs with respect Which of the pair(s) given above is/are
to Gulf of Guinea correctly matched?
1. Its coastline stretches in a huge arc from a. 1 only
Liberia to Gabon.
b. 2 only
2. It is located on the West African coast.
c. Both 1 and 2
Which of the statement(s) given above is/are
correct? d. Neither 1 nor 2

www.shankariasacademy.com | www.iasparliament.com
46

 Even though it is commonly used for


making pan masala for chewing, it has
Answers many medicinal, industrial and cultural
1. b usages.

 Tawlhlohpuan, a medium to heavy, compactly  It is considered as a remedy for bad breath


woven, good quality fabric from Mizoram is and digestive disorders.
known for warp yarns, warping, weaving &
 The Geographical Indication (GI) under the
intricate designs that are made by hand. Department for Promotion of Industry and
 Tawlhloh, in Mizo language, means 'to stand Internal Trade has recenly granted GI tag for
firm or not to move backward‘. PalaniPanchamirtham from Palani Town of
Tamil Nadu, Tawlhlohpuan and Mizo
 Tawlhlohpuan, which holds high significance Puanchei from the state of Mizoram, Tirur
in the Mizo society, is produced throughout the Betel leaf from Kerala.
state of Mizoram, Aizawl and Thenzawl town
being the main centre of production. 4. b

 The Geographical Indication (GI) under the  Mizo Puanchei, a colourful Mizo
Department for Promotion of Industry and shawl/textile, from Mizoram, is considered as
Internal Trade has recenly granted GI tag for the most colourful among the Mizo textiles.
PalaniPanchamirtham from Palani Town of  It is an essential possession for every Mizo lady
Tamil Nadu, Tawlhlohpuan and Mizo and an important marriage outfit in the state.
Puanchei from the state of Mizoram, Tirur
Betel leaf from Kerala.  It is also the most commonly used costume in
Mizo festive dances and official ceremonies.
2. d
 The weavers insert the designs and motifs by
 The CDS is a high military office that oversees using supplementary yarns while weaving to
and coordinates the working of the three create this beautiful and alluring textile.
Services.
 The Geographical Indication (GI) under the
 It offers seamless tri-service views and single- Department for Promotion of Industry and
point advice to the Executive (in India‘s Internal Trade has recenly granted GI tag for
case, to the Prime Minister) on long-term PalaniPanchamirtham from Palani Town of
defence planning and management, including Tamil Nadu, Tawlhlohpuan and Mizo
manpower, equipment and strategy, and above Puanchei from the state of Mizoram, Tirur
all, ―jointsmanship‖ in operations. Betel leaf from Kerala.
 India has had a feeble equivalent known as 5. b
the Chairman, Chiefs of Staff Committee
(CoSC); but this is a toothless office, given the  1945 Simla Conference was held
manner in which it is structured. between Viceroy Lord Wavell and major
political leaders in British India.
 At present, it is the National Security
Adviser (NSA) who advises India‘s Prime  The meeting was convened to seek approval of
Minister on military matters. Indian leaders for the Wavell Plan aimed to
usher in Indian self-rule.
3. d
 Tirur betel vine from Kerala, which is
mainly cultivated in Tirur, Tanur, Tirurangadi, 6. d
Kuttippuram, Malappuram and Vengara block 7. c
panchayaths of Malappuram District.
8. d
 It is valued both for its mild stimulant
action and medicinal properties. 9. d
10. d

www.shankariasacademy.com | www.iasparliament.com
47

 sanaa - Yemen a. 1 only


 Gibraltar - British Overseas Territory b. 2 only
c. Both 1 and 2
d. Neither 1 nor 2

3) Gond Art is a form of painting that is


practiced in which of the following states?
a. Nagaland
b. Tamil Nadu
c. Madhya Pradesh
d. Jammu and Kashmir

4) Which of the following countries


borders Baltic Sea?
1. Lithuania
2. Latvia
3. Estonia
19-08-2019 Select the correct answer using the codes given
1) Consider the following statements with below:
respect to Aadi Mahotsav 2019 a. 1 and 3 only
1. It is a joint initiative of Ministry of b. 1 and 2 only
Tribal Affairs and Tribal Cooperative
Marketing Development Federation of c. 1, 2 and 3
India (TRIFED). d. None
2. The theme for the festival is ―A
celebration of the spirit of Tribal Craft,
Culture and Commerce‖. 5) Okjokull glacier sometimes seen in the news
recently is located in?
Which of the statement(s) given above
is/are incorrect? a. Tibet
a. 1 only b. Greenland
b. 2 only c. Antarctica
c. Both 1 and 2 d. Iceland
d. Neither 1 nor 2
6) Which of the following countries is/are
members of BASIC Grouping?
2) Consider the following pairs
1. China
1. Jabro Dance – Manipur
2. India
2. Spawo dance – Mizoram
3. Brazil
Which of the pair(s) given above is/are
correctly matched? 4. Russia

www.shankariasacademy.com | www.iasparliament.com
48

5. Sri Lanka 9) Kurki is a practice banned in Punjab. It


refers to?
6. South America
a. It is the distribution of cannabis in religious
Select the correct answer using the codes given
occasions
below:
b. It is a form of shifting cultivation practiced in
a. 1, 2 and 3 only
North Western India
b. 1, 2, 3 and 6 only
c. It is the attachment of farmers‘ land by banks or
c. 2, 3 and 6 only traders in the event of non-payment of loan or
debt.
d. 1, 2, 3, 4 and 5 only
d. None of the above

7) Consider the following statements with


respect to NASA‟s Parker Solar Probe 10) Which of the following offices are the
Constitutional posts in India?
1. It is part of NASA‘s Living with a Star
programme that explores different 1. Special Officer for Linguistic Minorities
aspects of the Sun-Earth system.
2. Advocate General of State
2. The mission‘s central aim is to trace
3. Additional Solicitor General of India
how energy and heat move through the
Sun‘s corona and to study the source of Select the correct answer using the codes given
the solar wind‘s acceleration. below:
Which of the statement(s) given above is/are a. 1 and 2 only
correct?
b. 1 and 3 only
a. 1 only
c. 2 and 3 only
b. 2 only
d. 1, 2 and 3
c. Both 1 and 2
d. Neither 1 nor 2
Answers
1. d
8) Consider the following statements
 Adi Mahotsav is jointly organized by Tribal
1. Article 71 of the Constitution provides Affairs Ministry and TRIFED with a theme
for the option of a state to have a of A celebration of the spirit of Tribal
Legislative Council in addition to its Craft, Culture and Commerce.
Legislative Assembly.
2. d
2. As in Rajya Sabha, members of a
Legislative Council are also directly  The inaugural event of Aadi Mahotsav
elected by voters. (National Tribal Festival) that held recently
had two reputed local cultural troupes
Which of the statement(s) given above is/are presenting Ladakhi folk dances.
correct?
 The troupes performed Jabro Dance – a
a. 1 only nomadic dance and song of people inhabiting
b. 2 only eastern Ladakh and Spawo dance, a heroic
song and dance associated with a legendary
c. Both 1 and 2 hero of Himalayan region called K‘sar.
d. Neither 1 nor 2 3. c
4. c

www.shankariasacademy.com | www.iasparliament.com
49

8. d
 Article 169 empowers Parliament to provide
by law for the abolition or creation of the
Legislative Councils in States.
 As in Rajya Sabha, members of a Legislative
Council are not directly elected by voters.
9. c
 Two years after Punjab government notified a
ban on kurkis or attachment of farmers‟
land by banks in the event of their
defaulting on loan payments, several such
cases are taking place daily across districts in
the state‘s Malwa region.
 Kurki is attachment of farmers‘ land by banks
or arhtiyas/sahukars/traders (commission
agents and private money lenders) in the event
of non-payment of loan or debt.
10. a

20-08-2019
1) Consider the following statements with
respect to Comprehensive Convention on
International Terrorism (CCIT)
5. d 1. It was proposed by India in the United
Nations (UN) in 1996.
 In Iceland, people gather to commemorate
the loss of the glacier Okjokull, which was 2. It was ratified by U.N countries and
officially declared dead in 2014 at the age came in to force since 2014.
of 700. Which of the statement(s) given above is/are
 The glacier was officially declared dead by the correct?
Icelandic Meteorological Office when it was no a. 1 only
longer thick enough to move.
b. 2 only
 What once was glacier has been reduced to a
small patch of ice atop a volcano. c. Both 1 and 2

 Iceland loses about 11 billion tonnes of ice per d. Neither 1 nor 2


year, and scientists fear that all of the island
country's 400-plus glaciers will be gone by
2200. 2) Which of the following agencies is
responsible for command, control and
6. a operational decisions regarding India's nuclear
weapons programme?
 The BASIC countries are a grouping
of Brazil, South Africa, India and China. a. Strategic Forces Command
7. c b. Nuclear Command Authority
 On August 12, NASA’s Parker Solar Probe c. National Security Adviser
completed a year in service.
d. Department of Atomic Energy

www.shankariasacademy.com | www.iasparliament.com
50

b. Greenpeace
3) KONKAN is a bilateral exercise held c. UN Environment
between?
d. Centre for Science and Environment
a. India and Russia
b. India and USA
7) Ansupa Lake is located in which of the
c. India and UK following states?
d. India and Japan a. Odisha
b. Sikkim
4) Consider the following statements with c. Nagaland
respect to National Maritime Domain
d. Andhra Pradesh
Awareness (NMDA) Project
1. It was launched in accordance with the
objective of Project Mausam. 8) With respect to Lithuania, consider the
following statements
2. It was partially funded by World Bank.
1. It is a landlocked country located in the
Which of the statement(s) given above is/are
Baltic region.
correct?
2. It does not share its borders with
a. 1 only
Russia.
b. 2 only
Which of the statement(s) given above is/are
c. Both 1 and 2 correct?
d. Neither 1 nor 2 a. 1 only
b. 2 only
5) Consider the following statements with c. Both 1 and 2
respect to Biosimilarity sometimes seen in the
d. Neither 1 nor 2
news recently
1. They are identical versions of their
innovative biological medicine of 9) Consider the following pairs
reference whose patents have expired.
1. Riga – Poland
2. The practice of Biosimilarity is banned
in India since 2010. 2. Vilnius – Italy

Which of the statement(s) given above is/are Which of the pair(s) given above is/are
correct? correctly matched?

a. 1 only a. 1 only

b. 2 only b. 2 only

c. Both 1 and 2 c. Both 1 and 2

d. Neither 1 nor 2 d. Neither 1 nor 2

6) “Global SO2 emission hotspot 10) Which one of the following is not headed by
database” was recently published by which of the Prime Minister of India?
the following? a. Interstate council
a. NASA b. Nuclear Command Authority

www.shankariasacademy.com | www.iasparliament.com
51

c. National Ganga River Basin Authority minor differences in clinically inactive


components; and where there are no clinically
d. Zonal council
meaningful differences between the biological
product and the referenced product in terms of
safety, purity and potency.
Answers
 Biosimilars are “similar” but not
1. a identical versions of their innovative
 Comprehensive Convention on International biological medicine of reference whose patents
Terrorism (CCIT) is a draft proposed by India have expired.
in 1996 that has yet to be adopted by the  Dr. Reddy’s Laboratories has recently
UN General Assembly. launched in India Versavo (bevacizumab),
2. b a biosimilar of Roche’s Avastin for the
treatment of cancers.
 Nuclear Command Authority (NCA) is
India‘s nodal authority responsible for 6. b
command, control and operational decisions  India is the largest emitter of sulphur dioxide
regarding India‘s nuclear weapons (SO2) in the world, contributing more than 15
programme. per cent of global anthropogenic emissions,
 It has Executive Council and Political Council. according to a new report: Global SO2
emission hotspot
 The NCA‘s directives are executed by database by Greenpeace released recently.
the Strategic Forces Command.
 The primary reason for India‘s high emission
 The Executive Council is chaired by output is the expansion of coal-based
the National Security Adviser (NSA). electricity generation over the past decade, the
report added.
 It gives inputs to the Political Council, which
authorizes a nuclear attack if need be.  The analysis is based on hotspots detected
by NASA Ozone Monitoring Instrument
 The Political Council is chaired by the Prime
(OMI) satellite data that captured more than
Minister and is advised by the Executive
500 major source points of SO2 emissions
Council.
across the globe including natural sources such
3. c as volcanoes.
 It is an annual bilateral exercises  However, the analysis excluded all natural
between Indian Navy and Royal Navy of sources and only anthropogenic sources of SO2
Britain. were investigated.
 It is currently underway off the South Coast of 7. a
UK.
 The Odisha Wetland Authority has recently
 It aims to promote mutual understanding, approved implementation of an integrated
provide exposure to operating procedures, management plan for Chilika, country‘s
communication measures and best practices. largest brackish water lagoon, and Ansupa,
State‘s largest freshwater lake, at an estimated
4. d investment of Rs.180 crore.
 The National Maritime Domain Awareness 8. d
(NMDA) Project was launched in accordance
with the vision of Prime Minister Shri
Narendra Modi on SAGAR (Security And
Growth for All in the Region).
5. d
 Biosimilarity means ―highly similar‖ to the
referenced innovator product notwithstanding

www.shankariasacademy.com | www.iasparliament.com
52

2) Consider the following statements with


respect to National Real Estate Development
Council (NAREDCO)
1. It was established as an autonomous
self-regulatory body in 1998 under the
aegis of Ministry of Housing and Urban
Affairs.
2. Reserve Bank of India (RBI) contributed
the entire paid-up capital.
Which of the statement(s) given above is/are
correct?
a. 1 only
b. 2 only
c. Both 1 and 2
d. Neither 1 nor 2

3) NISHTHA is an initiative launched recently


by which of the following ministries?
a. Ministry of Corporate Affairs
b. Ministry of Science and Technology
9. d
c. Ministry of Health and Family Welfare
 Riga – Latvia
d. Ministry of Human Resources and
 Vilnius – Lithuania Development
10. d
4) Consider the following statements with
respect to Sericin
21-08-2019
1. It is a protein known to possess anti-
1) Consider the following statements with
oxidant and other medicinal properties.
respect to Incredible India Tourist Facilitator
Certification (IITFC) Programme 2. It can help make beauty and skincare
products.
1. It is an online programme where one
can learn about tourism at their own Which of the statement(s) given above is/are
time, space, path and pace. correct?
2. It is a digital initiative of Ministry of a. 1 only
Tourism.
b. 2 only
Which of the statement(s) given above
c. Both 1 and 2
is/are incorrect?
d. Neither 1 nor 2
a. 1 only
b. 2 only
5) Consider the following statements with
c. Both 1 and 2
respect to Anaemia of Inflammation
d. Neither 1 nor 2
1. It is mainly caused due to iron
deficiency.

www.shankariasacademy.com | www.iasparliament.com
53

2. It occurs due to shorter life span of red c. 2 and 3 only


blood cells in body.
d. 1, 2 and 3
Which of the statement(s) given above is/are
correct?
9) Bhakra Dam is built across which of the
a. 1 only
following?
b. 2 only
a. River Ganges
c. Both 1 and 2
b. River Indus
d. Neither 1 nor 2
c. River Sutlej
d. River Chenab
6) Tinospora cardiofolia commonly known as
gudduchi is a traditional herb used for
treating? 10) Asola Bhati Wildlife Sanctuary is located
in which of the following?
a. Inflammatory diseases
a. Delhi
b. Neurological disorders
b. Dadra & Nagar Haveli
c. Pancreatic Cancer
c. Puducherry
d. Tuberculosis
d. Lakshadweep

7) Consider the following statements with


respect to Rajiv Gandhi Khel Ratna Award Answers
1. It is the highest sporting honour of the 1. d
Republic of India.
 The Incredible India Tourist Facilitator
2. It is awarded annually by the Ministry of Certification (IITFC) Programme is a digital
Youth Affairs and Sports. initiative of Ministry of Tourism,
Which of the statement(s) given above is/are Government of India for the citizens of India to
correct? become a part of the booming Tourism
Industry.
a. 1 only
 It is an online programme where one can
b. 2 only learn about tourism at their own time, space,
c. Both 1 and 2 path and pace.
d. Neither 1 nor 2  The successful completion of this programme
would enable the learner to become a Certified
Tourist Facilitator of Ministry of Tourism,
8) Consider the following pairs Government of India.
Places in News – Country 2. a
1. San Nicolas Island – USA  National Real Estate Development
Council (NAREDCO) was established as an
2. Sanliurfa – Turkey autonomous self-regulatory body in 1998
3. Surabaya – Singapore under the aegis of Ministry of Housing
and Urban Affairs, Govt. of India.
Which of the pair(s) given above is/are
correctly matched?  It is the apex national body for the real estate
industry and visualized it as a single platform
a. 3 only where Government, industry and public would
b. 1 and 2 only discuss various problems and opportunities

www.shankariasacademy.com | www.iasparliament.com
54

face to face which would result in speedy as gudduchi or giloy could be useful in
resolution of issues. treating inflammation-induced anaemia.
3. d 6. a
 Union Ministry for Human Resources  Tinospora has been traditionally used
Development has recently launched the for inflammatory diseases but its
Integrated Teachers Training mechanism of action was not known which
Programme, NISHTHA National restricted its use.
Initiative for School Head‟s and
Teachers‟ Holistic Advancement, in New  Scientists in Pune have found that a traditional
Delhi. herb Tinospora cardiofolia commonly known
as gudduchi or giloy could be useful in
 This integrated programme aims to build the treating inflammation-induced anaemia.
capacities of around 42 lakh participants
7. c
covering all teachers and Heads of the school
at the elementary level in all government 8. b
school, faculty members of states councils of
Educational Research and Training and other  The U.S. military conducted a flight test of a
educational departments. conventionally-configured ground-launched
cruise missile recently in San Nicolas
 The integrated training will motivate and Island, USA.
equip teachers to encourage and foster critical
thinking in students, handle diverse situations  Sanliurfa, Turkey is a planned U.S-Turkey
and act as first-level counsellors. Joint headquarters to administer safe zone on
the Syria-Turkey border.
4. c
 Surabaya – Indonesia
 The protein, Sericin, is known to possess anti-
oxidant and other medicinal properties. 9. c

 These properties depend on amino acid  Bhakra Dam is a concrete gravity dam on
composition and secondary metabolites the Sutlej River in Bilaspur, Himachal
(polyphenols and flavonoids) of sericin. Pradesh in northern India.

 It can be used to develop a range of beauty and  The dam forms the Gobind Sagar reservoir.
skincare products. 10. a
5. b
 Anaemia is normally caused due to iron
deficiency, but cancers, autoimmune diseases,
infections and chronic kidney diseases may 22-08-2019
also lead to anaemia. 1) A report titled „Skin and Bones Unresolved:
 This is known as anaemia of An Analysis of Tiger Seizures from 2000-
inflammation. 2018‟ was published recently by?

 The inflammation-induced anaemia occurs a. TRAFFIC


due to shorter life span of red blood b. UN Environment
cells in body and also suppression in
formation of new red blood cells. c. Global Tiger Forum

 The currently available treatments for this type d. National Tiger Conservation Authority
of anaemia have several side effects. So
scientists have been looking for safer
medications. 2) Consider the following statements with
respect to South Atlantic Anomaly (SAA)
 Scientists in Pune have found that a traditional
herb Tinospora cardiofolia commonly known

www.shankariasacademy.com | www.iasparliament.com
55

1. It is a region above the South Atlantic 5) With respect to Citizenship of India,


Ocean where there are a large number consider the following statements
of charged particles.
1. Indian Constitution does not define the
2. It damages sensitive instruments; hence term ‗citizen‘.
all space telescopes are shut down when
2. Citizenship is under the exclusive
they pass through the SAA.
jurisdiction of Parliament.
Which of the statement(s) given above is/are
Which of the statement(s) given above is/are
correct?
correct?
a. 1 only
a. 1 only
b. 2 only
b. 2 only
c. Both 1 and 2
c. Both 1 and 2
d. Neither 1 nor 2
d. Neither 1 nor 2

3) Consider the following statements with


6) Quality Unknown: The Invisible Water
respect to Van Allen Radiation belt
Crisis is a report released recently by which of
1. These are donut-shaped rings the following?
surrounding our planet Earth.
a. World Bank
2. It is held by the planet‘s magnetic field
b. UN Water
around the planet.
c. UN Development Programme
3. Van Allen belts are not a part of our
atmosphere. d. World Health Organisation
Which of the statement(s) given above is/are
correct?
7) Arrange the following epoch in
a. 1 only chronological order
b. 1 and 2 only 1. Miocene
c. 2 and 3 only 2. Pliocene
d. 1, 2 and 3 3. Pleistocene
4. Holocene
4) Consider the following statements with Select the correct answer using the codes given
respect to State Rooftop Solar Attractiveness below:
Index–SARAL
a. 1-2-4-3
1. It evaluates Indian states based on their
attractiveness for rooftop development. b. 2-1-3-4

2. Andhra Pradesh tops the Index followed c. 1-3-2-4


by Telangana. d. 1-2-3-4
Which of the statement(s) given above is/are
correct?
8) Consider the following statements with
a. 1 only respect to TRAFFIC
b. 2 only 1. It is a non-governmental organisation
c. Both 1 and 2 working globally on trade in wild
animals and plants.
d. Neither 1 nor 2

www.shankariasacademy.com | www.iasparliament.com
56

2. It is headquartered in the United  India, with the world‘s largest wild tiger
Kingdom. population, topped in the trafficking of tigers
and tiger body parts over 19 years since 2000,
Which of the statement(s) given above is/are
a new TRAFFIC analysis from Geneva has
correct?
revealed.
a. 1 only
 The report titled „Skin and Bones
b. 2 only Unresolved: An Analysis of Tiger
c. Both 1 and 2 Seizures from 2000-2018‟ was the fourth
in a series on tiger trade by TRAFFIC, a
d. Neither 1 nor 2 wildlife trade monitoring network
headquartered in the United Kingdom.
2. c
9) Consider the following statements with
respect to CITES  The Earth is surrounded by a pair of concentric
1. It is an UN agreement adopted in 1965. donut-shaped clouds called the Van Allen
radiation belts which, like magnetic bottle,
2. Its aim is to ensure that international store and trap charged particles from the solar
trade in specimens of wild animals and wind.
plants does not threaten their survival.
 They are aligned with the magnetic axis of the
3. It is legally binding on its parties. Earth, which is tilted by 11 degrees from the
Which of the statement(s) given above is/are rotation axis of the Earth, and are not
correct? symmetrically placed with respect to the
Earth's surface.
a. 2 only
 Above South America, about 200 - 300
b. 1 and 2 only kilometers off the coast of Brazil, and
c. 2 and 3 only extending over much of South America, the
nearby portion of the Van Allen Belt forms
d. 1, 2 and 3 what is called the South Atlantic Anomaly.
 Satellites and other spacecraft passing through
10) Consider the following pairs this region of space actually enter the Van
Allen radiation belt and are bombarded by
Telescope – Country protons.
1. GROWTH – India  This can produce 'glitches' in astronomical
2. Kepler – USA data, problems with the operation of on-board
electronic systems, and premature aging of
Which of the pair(s) given above is/are computer, detector and other spacecraft
correctly matched? components.
a. 1 only  All space telescopes are shut down when they
b. 2 only pass through the SAA.

c. Both 1 and 2 3. d

d. Neither 1 nor 2  The Van Allen belts are not a part of our
atmosphere.
 They‘re well beyond it, extending hundreds of
Answers miles outwards into space.
1. a  A radiation belt refers to a layer of charged and
 India is the country with the highest overall energetic particles which is held by the planet‘s
number of seizures, says a study on wildlife magnetic field around the planet.
trade

www.shankariasacademy.com | www.iasparliament.com
57

 There are two, both donut-shaped Flora) is an international


rings surrounding our planet. agreement between governments.
 The main constituents of the belts are believed  Its aim is to ensure that international trade in
to come from solar wind and cosmic rays. specimens of wild animals and plants does not
threaten their survival.
 The belt is divided into two belts - outer and
inner.  Although CITES is legally binding on the
Parties – in other words they have to
 The outer belt of the radiation belt is formed implement the Convention – it does not take
by the energetic and charged electrons and the the place of national laws.
inner belt contains a combination of protons
and electrons and is formed as a result of  Rather it provides a framework to be respected
cosmic ray collisions in the upper atmosphere. by each Party, which has to adopt its own
domestic legislation to ensure that CITES is
 These particles do not have enough energy to implemented at the national level.
escape into space.
10. c
4. a
1. GROWTH – India
 The Union Minister of State for Power and
New & Renewable Energy (IC) and Skill 2. Kepler – USA
Development & Entrepreneurship has recently
launched the State Rooftop Solar
Attractiveness Index-SARAL. 23-08-2019
 The State of Karnataka has been placed at 1) Starting from the pretty hamlet of Sarbal,
the first rank in the Index that evaluates one has to hike up to the Durinar meadows to
Indian states based on their attractiveness for reach the Durinar lake — a crystal clear blue
rooftop development. lake that is safely concealed among giant
granite spires. The Nilnag lake lies further up
 Telangana ranks second in the index. in the Thejwas range. From here on, trudging
5. c over steep snowy slopes and ice fields of the
Kolohai glacier one would reach Barafsar
 Citizenship is in the Union List under the (frozen lake). From this vantage point one can
Constitution and thus under the exclusive see the fascinating 5,425 m tallest peak, Mt.
jurisdiction of Parliament. Kolohai (Gwash Brari or the Goddess of light).
 The Constitution does not define the term Which of the following States has been
„citizen‟ but gives, in Articles 5 to 11, details of depicted in the above paragraph?
various categories of persons who are entitled
to citizenship. a. Sikkim

 Unlike other provisions of the Constitution, b. Himachal Pradesh


which came into being on January 26, 1950, c. Arunachal Pradesh
these articles were enforced on November 26,
1949 itself, when the Constitution was d. Jammu and Kashmir
adopted.
6. a 2) Consider the following statements with
7. d respect to Fedor (Final Experimental
Demonstration Object Research)
8. c
1. It is an airborne observatory that aims
9. c to integrate and improve the level of
education in various sectors dealing
 CITES (the Convention on International Trade
with science and technology.
in Endangered Species of Wild Fauna and

www.shankariasacademy.com | www.iasparliament.com
58

2. It is an initiative of National 2. Central banks that are not members of


Aeronautics and Space the Bank of International Settlements
Administration (NASA). are not allowed to register as FPIs and
invest in India under the new norms.
Which of the statement(s) given above is/are
correct? Which of the statement(s) given above is/are
correct?
a. 1 only
a. 1 only
b. 2 only
b. 2 only
c. Both 1 and 2
c. Both 1 and 2
d. Neither 1 nor 2
d. Neither 1 nor 2

3) Consider the following statements with


respect to Sabka Vishwas (Legacy Dispute 6) Consider the following statements with
Resolution) Scheme respect to Battle of Adyar
1. The Scheme is especially tailored to free 1. It was fought between the French East
the large number of small taxpayers of India Company men and Nawab of Arcot
their pending disputes with the tax forces over the St. George Fort.
administration.
2. It was part of the Second Carnatic War
2. The two main components of the between the English and the French.
Scheme are dispute resolution and
Which of the statement(s) given above is/are
amnesty.
correct?
Which of the statement(s) given above is/are
a. 1 only
correct?
b. 2 only
a. 1 only
c. Both 1 and 2
b. 2 only
d. Neither 1 nor 2
c. Both 1 and 2
d. Neither 1 nor 2
7) Consider the following statements with
respect to National Archives of India
4) HR Khan Committee sometimes seen in the
1. It is a repository of non-current records
news recently is associated with which of the
of government of India and holds them
following?
in trust for the use of administration.
a. Foreign Portfolio Investors (FPI) norms
2. It is an attached office under the
b. National Coal Mine Policy Ministry of Culture.
c. New direct tax legislation Which of the statement(s) given above is/are
correct?
d. None of the above
a. 1 only
b. 2 only
5) Consider the following statements with
respect to Foreign Portfolio Investors (FPI) c. Both 1 and 2
norms which was relaxed recently
d. Neither 1 nor 2
1. SEBI has recently simplified the
registration process for FPIs by doing
away with the broad-based eligibility 8) Consider the following statements with
criteria. respect to Project SURE

www.shankariasacademy.com | www.iasparliament.com
59

1. It aims to identify, recognise and  In the Union Budget 2019-20, Union Finance
connect lost children with their family. Minister announced the Sabka Vishwas-
Legacy Dispute Resolution Scheme,
2. It was launched by the Ministry of Home
2019.
Affairs.
Which of the statement(s) given above is/are  The two main components of the Scheme
correct? are dispute resolution and amnesty.

a. 1 only  The Scheme is especially tailored to free the


large number of small taxpayers of their
b. 2 only pending disputes with the tax administration.
c. Both 1 and 2 4. a
d. Neither 1 nor 2  SEBI had constituted a working committee
under the chairmanship of HR Khan, retired
deputy governor, Reserve Bank of India, which
9) The second round of Composite Water was entrusted with the task of reviewing the
Management Index 2.0 will be released by current Securities and Exchange Board of
which of the following? India (Foreign Portfolio Investors)
a. NITI Aayog Regulations, 2014 and recommending any
amendments that may be required for
b. Ministry of Jal Sakthi rationalising and simplifying the SEBI FPI
c. Ministry of Urban development regulations.

d. Ministry of Rural development 5. a


 The Securities and Exchange Board of India
(SEBI), based on the recommendations of the
10) Baltal-Thajwas Wildlife Sanctuary is H.R. Khan committee, eased several regulatory
located in which of the following states? restrictions.
a. Jammu and Kashmir  Among a slew of measures, the financial
b. Arunachal Pradesh markets regulator has simplified the
registration process for FPIs by doing away
c. Manipur with the broad-based eligibility criteria,
d. Madhya Pradesh which required a minimum of at least 20
investors in a foreign fund, and certain
documentary requirements.
Answers  Central banks that are not members of the
Bank of International Settlements are also
1. d
allowed to register as FPIs and invest in the
2. d country under the new norms.
 Russia has recently launched an unmanned 6. a
rocket carrying a life-size humanoid robot that
 The Battle of Adyar took place on 24 October
will spend 10 days learning to assist astronauts
1746.
on the International Space Station.
 The battle was between the French East
 Named Fedor, short for Final
India Company men and Nawab of Arcot
Experimental Demonstration Object
forces over the St. George Fort, which was
Research, the robot is the first ever sent up
held by the French.
by Russia.
 It was part of the First Carnatic
 Fedor is also known as Skybot F850.
War between the English and the French.
3. c

www.shankariasacademy.com | www.iasparliament.com
60

7. c Which of the statement(s) given above is/are


correct?
8. d
a. 1 only
 The SU.RE project is a commitment by India‘s
apparel industry to set a sustainable pathway b. 2 only
for the Indian fashion industry.
c. Both 1 and 2
 It was launched by the Ministry of Textiles. d. Neither 1 nor 2
 SU.RE stands for „Sustainable
Resolution‟ – a firm commitment from the
industry to move towards fashion that 3) Anubhav‟ portal sometimes seen in the news
contributes to a clean environment. recently was established under which of the
following ministries?
9. a
a. Ministry of Skill Development and
10. a Entrepreneurship
b. Ministry of Personnel, Public grievances and
24-08-2019 Pensions

1) Consider the following statements with c. Ministry of Social Justice and Empowerment
respect to Report on the Composite Water d. Ministry of Human Resources and
Management Index 2.0 Development
1. It ranks various states for the reference
year 2017-18 as against the base year
2016-17. 4) Mission Reach Out sometimes seen in the
news recently is associated with which of the
2. Gujarat tops the index in the Non- following regions?
Himalayan states category followed by
Andhra Pradesh. a. Kerala

3. It was prepared by NITI Aayog in b. Odisha


association with Ministry of Jal Shakti c. Maharashtra
and Ministry of Rural Development.
d. Jammu and Kashmir
Which of the statements given above are
correct?
a. 1 and 2 only 5) Zayed Medal is the top civilian honour of?

b. 2 and 3 only a. Qatar

c. 1, 2 and 3 b. Pakistan

d. None c. Saudi Arabia


d. United Arab Emirates

2) Consider the following statements with


respect to Fortification of Rice and its 6) Consider the following statements with
Distribution under Public Distribution System respect to „San-Sadhan‟ Hackathon Iniative
Scheme
1. It is an initiative to ease lives of Persons
1. Under the scheme rice is fortified with with Disabilities (Divyangjan) by
Iron, Folic Acid and Vitamin B-12. making toilets smarter, more accessible,
2. It is a centrally sponsored scheme and easier to use.
launched by the Ministry of Consumer 2. It is being organized jointly by the
Affairs, Food and Public Distribution Ministry of Jal Shakti and the
across the country.

www.shankariasacademy.com | www.iasparliament.com
61

Department of Empowerment of 1. These councils are setup for the welfare


Persons with Disabilities. of tribes only in the states under the
Fifth and Sixth Schedule areas
Which of the statement(s) given above is/are
correct? 2. These councils are setup by the order of
the President
a. 1 only
Which of the given above statement(s) is/are
b. 2 only
correct?
c. Both 1 and 2
a. 1 only
d. Neither 1 nor 2
b. 2 only
c. Both 1 and 2
7) Consider the following statements with
d. Neither 1 nor 2
respect to Zonal Councils
1. Zonal Councils are statutory bodies
established by an Act of the Parliament. 10) Which of the following reports/indices are
prepared by the United Nation Organization?
2. Its one of the main objective is to
achieve an emotional integration of the 1. Gender Inequality Index
country.
2. World Happiness Report
Which of the statement(s) given above is/are
3. Global Environment Outlook
correct?
Select the correct answer using the code given
a. 1 only
below:
b. 2 only
a. 3 only
c. Both 1 and 2
b. 1 and 2 only
d. Neither 1 nor 2
c. 2 and 3 only
d. 1, 2 and 3
8) Which of the following is/are various forms
of Ocean Energy?
1. Wave Energy Answers
2. Tidal Energy 1. c

3. Ocean Thermal Energy Conversion 2. a

4. Ocean Current Energy  Government of India has approved


the Centrally Sponsored Pilot Scheme on
Select the correct answer using the codes given ―Fortification of Rice and its Distribution
below: under Public Distribution System‖ on
a. 1 and 2 only 14.02.2019 - rice to be fortified with Iron,
Folic Acid and Vitamin B-12.
b. 3 and 4 only
 The Pilot Scheme has been approved for a
c. 1, 2 and 4 only period of three year beginning 2019-20.
d. 1, 2, 3 and 4  The Pilot Scheme at present focuses on 15
districts during the initial phase of
implementation.
9) Consider the following statements
regarding Tribes Advisory Council  The operational responsibilities for
implementation of the Pilot Scheme lie with
the States/UTs.

www.shankariasacademy.com | www.iasparliament.com
62

3. b  Tribal Advisory councils are not


associated with Sixth Schedule areas.
 It is a platform for retirees to share experience
of working with the Government.  These councils are set up by the order of
the Governor.
 This facility provides a sense of fulfilment and
satisfaction to the retirees and also creates a 10. d
data base of useful suggestion and
information.
 An annual award scheme has been instituted 26-08-2019
to encourage the retiree to submit his valuable 1) Microplastics in drinking water Report was
experience/ suggestions through write-ups. recently published by which of the following?
 Union Minister for Personnel, Public a. World Health Organisation
Grievances and Pensions has recently
presented the ANUBHAV awards, 2019. b. UN Water

4. d c. UN Environment
d. Greenpeace International
 The Indian Army has launched "Mission
Reach Out" in Jammu to ensure basic
necessities and essential services are available
in the region post the abrogation of the 2) Consider the following statements with
provisions of Article 370 and reorganisation respect to Special Protection Group (SPG)
of Jammu and Kashmir. 1. It was raised with the intention to
5. d provide proximate security cover to the
Prime Minister, former Prime Ministers
 Indian Prime Minister Narendra Modi was and their immediate family members.
recently awarded the United Arab
2. It works under the Ministry of Defence.
Emirate's (UAE) highest civilian
honour Zayed medal. Which of the statement(s) given above is/are
correct?
6. c
a. 1 only
 The government calls for applications for its
latest initiative under the Swachh Bharat b. 2 only
Mission, called the ‗San-Sadhan‘ Hackathon,
c. Both 1 and 2
an initiative to ease lives of Persons with
Disabilities (Divyangjan) by making toilets d. Neither 1 nor 2
smarter, more accessible, and easier to use.
 In this hackathon, the government is looking
3) Consider the following statements with
for smart, scalable and innovative solutions for
respect to ASRAAM sometimes seen in the
economical toilets for individual and
news recently
community use in rural and urban contexts.
1. It is an advanced short-range air-to-air
 The initiative is being organized jointly by missile.
the Ministry of Jal Shakti and
the Department of Empowerment of 2. It is widely used as a Within Visual
Persons with Disabilities, in collaboration Range (WVR) air dominance missile
with Atal Innovation Mission, NITI Aayog, with a range of over 25km.
Bill & Melinda Gates Foundation, and
3. It will be the first wing-launched missile
91springboard.
in the Indian Air Force inventory.
7. c
Which of the statement(s) given above is/are
8. d correct?
9. d a. 3 only

www.shankariasacademy.com | www.iasparliament.com
63

b. 1 and 2 only 7) Consider the following statements with


respect to Special Data Dissemination
c. 2 and 3 only
Standard (SDDS)
d. 1, 2 and 3
1. It is an initiative launched by the
International Monetary Fund (IMF).
4) Annual Observance Report of the Special 2. It is mandatory for all IMF members.
Data Dissemination Standard for 2018 was
3. India subscribed to the SDDS in 1996.
released recently by?
Which of the statement(s) given above is/are
a. World Bank
correct?
b. UN Statistics Division
a. 3 only
c. World Economic Forum
b. 1 and 3 only
d. International Monetary Fund
c. 1, 2 and 3
d. None
5) Consider the following statements with
respect to Advisory Board for Banking Frauds
(ABBF) 8) Consider the following statements with
respect to Press Council of India (PCI)
1. It was constituted by the Ministry of
Finance in consultation with the 1. It is an autonomous, statutory, quasi-
Reserve Bank of India (RBI). judicial body which acts as the watchdog
of the press.
2. It was mandated to examine bank fraud
of over Rs. 50 crore and recommend 2. It is headed by a Chairman, who has by
action. convention, been a retired judge of the
Supreme Court of India.
3. It would function as the last level of
examination of all large fraud cases Which of the statement(s) given above is/are
before recommendation. correct?
Which of the statement(s) given above is/are a. 1 only
correct?
b. 2 only
a. 2 only
c. Both 1 and 2
b. 1 and 2 only
d. Neither 1 nor 2
c. 1, 2 and 3
d. None
9) Consider the following pairs
Places in News - Region
6) Air quality monitoring instruments in India
1. Tsuen Wan – Hong Kong
will be certified by which of the following
agencies? 2. Porto Velho – Chile
a. Bureau of Indian Standards Which of the pair(s) given above is/are
correctly matched?
b. Indian Standards Institute
a. 1 only
c. CSIR – National Physical Laboratory
b. 2 only
d. None of the above
c. Both 1 and 2
d. Neither 1 nor 2

www.shankariasacademy.com | www.iasparliament.com
64

6. c
10) Which one among the following was the  The Union Environment Ministry has
first country to declare ‗Climate Emergency‘? recently tasked the Council of Scientific &
Industrial Research (CSIR)-National
a. Sweden
Physical Laboratory (NPL) with
b. Ireland certifying air quality monitoring instruments.
c. New Zealand 7. c
d. United Kingdom  The International Monetary Fund (IMF)
launched the Special Data Dissemination
Standard (SDDS) initiative in 1996 to guide
Answers members to enhance data transparency and
help financial market participants with
1. a adequate information to assess the economic
2. a situations of individual countries.

 It works under the Cabinet Secretariat.  India subscribed to the SDDS on December
27, 1996.
3. d
 SDDS is a practice mandatory for all IMF
 Advanced Short-range Air-to-Air members.
Missile (ASRAAM)
 It was launched to guide members to enhance
 The ASRAAM is widely used as a within visual data transparency and help financial market
range (WVR) air-dominance missile with a participants with adequate information to
range of over 25 km. assess the economic situations of individual
 It will be the first wing-launched countries.
missile in the IAF inventory. 8. c
 All missiles are now fired from under the wing. 9. a
 The IAF is looking to adopt the ASRAAM on its 1. Tsuen Wan – Hong Kong
Su-30MKIs and the indigenous light combat
2. Porto Velho – Brazil
aircraft.
10. d
4. d
5. a
 The Central Vigilance Commission 27-08-2019
(CVC) has recently constituted an Advisory 1) Consider the following statements with
Board for Banking Frauds respect to C-Sat-Fi technology
(ABBF) headed by former Vigilance
Commissioner T.M. Bhasin to examine bank 1. It is based on the optimal utilization of
fraud of over Rs. 50 crore and recommend wireless and satellite communication to
action. extend connectivity to the unserved
areas.
 The panel in its previous avatar was called
the Advisory Board on Bank, 2. It does not require Satellite Phones and
Commercial and Financial Frauds. can work on any WiFi enabled phone.

 The ABBF, formed in consultation with Which of the statement(s) given above is/are
the RBI, would function as the first level of correct?
examination of all large fraud cases before a. 1 only
recommendations or references are made to
the investigative agencies by the respective b. 2 only
public sector banks (PSBs). c. Both 1 and 2

www.shankariasacademy.com | www.iasparliament.com
65

d. Neither 1 nor 2 Select the correct answer using the codes given
below:
a. 1-c; 2-a; 3-b
2) Consider the following statements with
respect to Group of Seven (G7) b. 1-a; 2-b; 3-c
1. It is an intergovernmental organisation c. 1-b; 2-c; 3-a
formed as an informal forum to discuss
d. 1-c; 2-b; 3-a
pressing world issues.
2. The G7 nations meet at annual Summits
that are presided over by leaders of 5) Consider the following statements with
member countries on a rotational basis. respect to Tokay gecko
3. India participated in the 45th annual 1. They are nocturnal lizards.
summit of G7 held in Biarritz, France.
2. They are recently included for the first
Which of the statement(s) given above is/are time under Appendix II of CITES.
correct?
3. It was listed as least concern under the
a. 3 only IUCN Red list.
b. 1 and 2 only Which of the statements given above are
correct?
c. 2 and 3 only
a. 1 and 2 only
d. 1, 2 and 3
b. 2 and 3 only
c. 1, 2 and 3
3) Consider the following statements with
respect to Smooth-coated Otter d. None
1. It is distributed throughout south Asia
and south-east Asia.
6) Mitra Crater sometimes seen in the news
2. It has been listed as vulnerable under recently is associated with which of the
the IUCN Red list. following?
Which of the statement(s) given above is/are a. Jupiter
correct?
b. Mars
a. 1 only
c. Antarctica
b. 2 only
d. None of the above
c. Both 1 and 2
d. Neither 1 nor 2
7) Consider the following statements with
respect to Gravitational Lensing
4) Match the following with respect 1. It occurs when a huge amount of matter
to CITES Agreement creates a gravitational field that distorts
and magnifies the light from objects
1. Appendix I – a. Species not necessarily
behind it.
threatened with extinction
2. It is useful to cosmologists because it is
2. Appendix II – b. Species that are
directly sensitive to the amount and
protected in at least one country
distribution of dark matter.
3. Appendix III – c. Species threatened
Which of the statement(s) given above is/are
with extinction
correct?
a. 1 only

www.shankariasacademy.com | www.iasparliament.com
66

b. 2 only disasters and emergencies when no other


means of communication are available, he
c. Both 1 and 2
added.
d. Neither 1 nor 2
 This cost-effective solution does not require
the expensive Satellite Phones and can
work on any WiFi enabled phone.
8) Consider the following statements with
respect to Chippiparai dogs 2. d
1. It is an ace runner and hunter.  The G7 or ‗Group of Seven‘ are Canada,
2. It is the native breed of Southern France, Germany, Italy, Japan, the United
Kerala. Kingdom, and the United States.

Which of the statement(s) given above is/are  It is an intergovernmental organisation that


correct? was formed in 1975 by the top economies of
the time as an informal forum to discuss
a. 1 only pressing world issues.
b. 2 only  The G7 nations meet at annual Summits that
c. Both 1 and 2 are presided over by leaders of member
countries on a rotational basis.
d. Neither 1 nor 2
 The current G7 Summit being held
in France is the 45th, and the next one will
9) Open Acreage Licensing is associated with be held in the United States in 2020.
which of the following sector?  The host country typically gets to invite
a. Polymetallic nodules dignitaries from outside the G7 to attend the
Summit.
b. Hydrocarbon exploration
 India was invited to attend the Summit as a
c. Highway Construction special guest of French President Emmanuel
d. None of the above Macron.
3. c
10) Which one of the following region has the  The Smooth-coated Otter is distributed
highest annual range of temperature? throughout south Asia and south-east
Asia.
a. Savannah climate
 Its distribution is continuous from Indonesia,
b. Siberian climate through south-east Asia, and westwards from
c. China climate southern China to India and Pakistan, with an
isolated subpopulation in Iraq.
d. Hot Desert climate
 Members at the CITES CoP 18 Conference
have voted to move the smooth-coated
Answers otter (Lutrogale perspicillata) from CITES
Appendix II to CITES Appendix I ―because it is
1. c considered to be facing a high risk of extinction
 C-Sat-Fi (C-DOT Satellite WiFi) is based and is detrimentally affected by international
on the optimal utilization of wireless and trade, as well as habitat loss and degradation
satellite communication to extend connectivity and persecution associated with conflict with
to the unserved areas including the remote people (and fisheries)‖.
islands and difficult terrains. 4. a
 Besides offering the ease of deployment, the  Members at the CITES CoP 18 Conference
solution is ideally suited to addressing have voted to move the smooth-coated

www.shankariasacademy.com | www.iasparliament.com
67

otter (Lutrogale perspicillata) from CITES  The phenomenon occurs when a huge amount
Appendix II to CITES Appendix I ―because it is of matter, such as a massive galaxy or cluster
considered to be facing a high risk of extinction of galaxies, creates a gravitational field that
and is detrimentally affected by international distorts and magnifies the light from objects
trade, as well as habitat loss and degradation behind it, but in the same line of sight.
and persecution associated with conflict with
people (and fisheries)‖.  Gravitational lensing is useful to cosmologists
because it is directly sensitive to the amount
 The other proposal that was passed was to and distribution of dark matter.
include the Tokay gecko (Gekko gecko) in
CITES Appendix II.  This is because the amount of light bending is
sensitive only to the strength of the
Appendix of CITES gravitational field it passes through, which is
mostly generated by the mass of the dark
 Appendix I includes species ―threatened
matter in the Universe.
with extinction‖; according to the CITES
website, ―trade in specimens of these species is 8. a
permitted only in exceptional circumstances‖.
 The Chippiparai breed of dogs belongs to
 Appendix II provides a lower level of the Tamil Nadu region of South India.
protection.
 A few of them also reside in Kerela, around
 There is also an Appendix III, which Periyar Lake.
―contains species that are protected in
at least one country, which has asked other  Chippiparai dogs are an ace runner and
CITES Parties for assistance in controlling the hunter.
trade‖. 9. b
5. c 10. b
 Tokay gecko (Gekko gecko) is nocturnal  The annual range is defined as the difference
lizards. between the hottest and coldest months, taking
 They are included for the first time monthly mean temperatures in each case.
under Appendix II.  The climate of Siberia varies dramatically, but
 It was listed as least concern under the all of it basically has short summers and long
IUCN Red list. and extremely cold winters.

 They are used in Chinese traditional medicine


and traded throughout south-east Asia in dried 28-08-2019
form or preserved in alcohol.
1) Consider the following statements with
6. d respect to Bonn Challenge
 Indian Space Research Organisation (ISRO) 1. It is a global effort to bring 150 million
has recently released the images of the lunar hectares of the world‘s deforested and
surface captured from the Chandrayaan-2 degraded land into restoration by 2030,
spacecraft orbiting the Moon. and 350 million hectares by 2040.
 The images clicked from an altitude of 4,375 2. At the UNFCC Conference of the Parties
Km also shows a crater – Mitra, which was (COP) 2015 in Paris, India joined the
named after Professor Sisir Kumar Mitra, an voluntary Bonn Challenge.
Indian physicist and Padma Bhushan recipient
who was known for his pioneering work in the Which of the statement(s) given above is/are
field of ionosphere and Radiophysics. correct?

7. c a. 1 only
b. 2 only

www.shankariasacademy.com | www.iasparliament.com
68

c. Both 1 and 2 4) Consider the following statements with


respect to United Nations Convention to
d. Neither 1 nor 2
Combat Desertification (UNCCD)
1. It is one among the three Rio
2) Consider the following statements with Conventions that resulted in the Earth
respect to National Monuments Authority Summit, 1992.
(NMA)
2. India is a signatory to UNCCD which is
1. It is a statutory body established by an the only legally binding international
Act of Parliament. agreement linking environment and
development issues to the land agenda.
2. It is the premier organization for the
protection of the cultural heritage of the 3. 14th session of the Conference of Parties
nation and archaeological researches. (COP-14) of the UNCCD will be hosted
by Kenya in Nairobi.
3. It works under the Ministry of Culture.
Which of the statement(s) give above is/are
Which of the statement(s) given above is/are correct?
correct?
a. 1 only
a. 3 only
b. 1 and 2 only
b. 1 and 3 only
c. 1 and 3 only
c. 2 and 3 only
d. 2 and 3 only
d. 1, 2 and 3

5) Keezhadi often seen in the news recently is a


3) Arrange the following in correct sequence major archaeological site located in the state
with respect to Hurricane formation of?
1. When storm passes over the land, a. Kerala
supply of moisture and heat is cut off
and storm gradually dissipates b. Odisha
2. Clouds form in the upper atmosphere as c. Tamil Nadu
the warm air condenses
d. None of the above
3. Wind spirals upwards and outwards
and a low pressure system forms over
ocean surface 6) Consider the following statements with
respect to Cyber Swachhta Kendra
4. Entire storm system starts to spin,
anticlockwise in the southern part of the 1. Its objective is to enhance the cyber
globe security of Digital India's IT
infrastructure by providing information
5. Warm water evaporates, moisture rises
on botnet/malware threats and
forming thunderstorms
suggesting remedial measures.
Select the correct answer using the codes given
2. It was launched by the Ministry of Home
below:
Affairs.
a. 5-2-3-4-1
Which of the statement(s) given above is/are
b. 2-5-4-1-3 correct?
c. 5-3-2-4-1 a. 1 only
d. 5-3-4-2-1 b. 2 only
c. Both 1 and 2

www.shankariasacademy.com | www.iasparliament.com
69

d. Neither 1 nor 2 d. Scientific Research

7) Consider the following statements with 10) Consider the following pairs
respect to Indian Cybercrime Coordination
Places in News – Country
Centre (I4C) Scheme
1. Brasilia – Brazil
1. The Scheme acts as a nodal point in the
fight against cybercrime and was 2. Kazan – Afghanistan
proposed only for two years (2018-
2020). Which of the pair(s) given above is/are
correctly matched?
2. It was launched by the Ministry of
Electronics and Information Technology a. 1 only
(MeitY). b. 2 only
Which of the statement(s) given above is/are c. Both 1 and 2
correct?
d. Neither 1 nor 2
a. 1 only
b. 2 only
Answers
c. Both 1 and 2
1. b
d. Neither 1 nor 2
 The Bonn Challenge is a global effort to bring
150 million hectares of the world‘s deforested
8) Consider the following statements with and degraded land into restoration by 2020,
respect to WorldSkills Competition and 350 million hectares by 2030.
1. WorldSkills hosts the worldskills  At the UNFCC Conference of the Parties (COP)
competition for every two years. 2015 in Paris, India also joined the voluntary
Bonn Challenge pledge to bring into
2. The 45th worldskills competition was restoration 13 million hectares of degraded
recently held in Aix-en-Provence, and deforested land by the year 2020, and
France. additional 8 million hectares by 2030.
3. India participated in the event and
 India‘s pledge is one of the largest in Asia.
finished 13th out of 63 countries.
2. b
Which of the statement(s) given above is/are
correct?  The Archaeological Survey of India
a. 3 only (ASI), under the Ministry of Culture, is the
premier organization for the archaeological
b. 1 and 2 only researches and protection of the cultural
heritage of the nation.
c. 1 and 3 only
3. c
d. 1, 2 and 3
How a Hurricane Forms?
1. Warm water evaporates, moisture rises
9) Tenzing Norgay Awards are given every
forming thunderstorms
year to recognize the achievements of persons
in the fields of? 2. Wind spirals upwards and outwards and a low
pressure system forms over ocean surface
a. Adventure
3. Clouds form in the upper atmosphere as the
b. Literature
warm air condenses
c. Sports

www.shankariasacademy.com | www.iasparliament.com
70

4. Entire storm system starts to spin,  The "Cyber Swachhta Kendra" (Botnet
anticlockwise in the southern part of the globe Cleaning and Malware Analysis Centre) is a
part of the Indian Computer Emergency
5. When storm passes over the land, supply of
Response Team (CERT-In).
moisture and heat is cut off and storm
gradually dissipates  It has been set up for analyzing BOTs/malware
4. b characteristics and providing information and
enabling citizens for removal of
 Established in 1994, the Convention entered BOTs/malware.
into force in December 1996.
 In addition, "Cyber Swachhta Kendra" will
 It is one of the three Rio strive to create awareness among citizens to
Conventions along with United Nations secure their data, computers, mobile phones
Framework Convention on Climate Change and devices such as home routers.
(UNFCCC) and Convention on Biological
7. a
Diversity (CBD).
 Ministry of Home Affairs (MHA) has
 UNCCD is the only legally binding
rolled out a scheme ‗Indian Cyber Crime
international agreement linking environment
Coordination Centre (I4C)‘ for the
and development issues to the land agenda.
period 2018-2020, to combat cyber crime in
 India became a signatory to UNCCD on 14th the country, in a coordinated and effective
October 1994 and ratified it on 17th December manner.
1996.
I4C Scheme
 The main objective of the convention is to
 To act as a nodal point in the fight against
combat desertification and mitigate the effects
cybercrime
of drought in countries experiencing serious
drought and/or desertification, involving long-  Identify the research problems/needs of LEAs
term integrated strategies that focus and take up R&D activities in developing new
simultaneously, in affected areas, on improved technologies and forensic tools in collaboration
productivity of land, and the rehabilitation, with academia / research institutes within
conservation and sustainable management of India and abroad
land and water resources, leading to improved
living conditions, in particular at the  The scheme is proposed with an outlay of Rs.
community level. 415.86 Crore and will run for two years

 India for the first time will host the 14th  To prevent misuse of cyber space for
session of the Conference of Parties (COP-14) furthering the cause of extremist and terrorist
of the United Nations Convention to Combat groups
Desertification (UNCCD) in September 2019.  Suggest amendments, if required, in cyber laws
5. c to keep pace with fast changing technologies
and International cooperation
 After wall structures and ring well, the State
Department of Archaeology has found a tank-  To coordinate all activities related to
like brick structure and terracotta pipelines implementation of Mutual Legal Assistance
during the excavation at Keezhadi. Treaties (MLAT) with other countries related
to cybercrimes in consultation with the
 Keezhadi is a major archaeological site concerned nodal authority in MHA
located in the state of Tamil Nadu.
Components of the scheme:
6. a
1. National Cybercrime Threat Analytics Unit.
 Cyber Swachhta Kendra is a part of the
Government of India's Digital India initiative 2. National Cybercrime Reporting Portal.
under the Ministry of Electronics and 3. Platform for Joint Cybercrime Investigation
Information Technology (MeitY). Team.

www.shankariasacademy.com | www.iasparliament.com
71

4. National Cybercrime Forensic Laboratory Which of the statement(s) given above is/are
Ecosystem. correct?
5. National Cybercrime Training Centre. a. 1 only
6. Cybercrime Ecosystem Management Unit. b. 2 only
7. National Cyber Research and Innovation c. Both 1 and 2
Centre.
d. Neither 1 nor 2
8. c
 Every two years WorldSkills hosts the world
2) ―Status of Policing in India Report
skills competition which attracts more than
2019” was recently published by which of the
1,300 Competitors from more than 60
following?
countries.
a. NITI Aayog
 The 45th worldskills competition was recently
held in Kazan, Russia. b. Common Cause
 India participated in the event and finished c. Ministry of Home Affairs
13th out of 63 countries. d. None of the above
9. a
 Tenzing Norgay National Adventure Awards 3) With respect to Shagun, consider the
are given every year to recognize the following statements
achievements of persons in the fields of
adventure, to encourage young people to 1. It aims to improve school education
develop the spirit of endurance, risk–taking, system by creating a junction for all
cooperative teamwork and quick, ready and online portals and websites relating to
effective reflexes in challenging situations and various activities of the Department of
to provide incentive to the young people for School Education and Literacy in the
getting exposed to the adventure activities. Government of India and all States and
Union Territories.
 The award is given in four categories namely,
Land Adventure, Water Adventure, Air 2. It was launched by the Ministry of
Adventure and Life Time Achievement. Human Resources and Development.

10. a Which of the statement(s) given above is/are


correct?
1. Brasilia – Brazil
a. 1 only
2. Kazan – Russia
b. 2 only
c. Both 1 and 2
29-08-2019
d. Neither 1 nor 2
1) Consider the following statements with
respect to Biometric Seafarer Identity
Document (BSID) 4) Convention No. 185 of the International
Labour Organisation (ILO) deals with?
1. It will give a foolproof identification to
seafarers which will facilitate their a. Poisoning Arising from Benzene
movement, provide ease of getting jobs
and help in identifying them from any b. Equality of Treatment (Social Security)
location in the world. c. Seafarer Identity Document
2. India is the first country in the world to d. Protection of Workers against Ionising
issue Biometric Seafarer Identity Radiations
Document (BSID).

www.shankariasacademy.com | www.iasparliament.com
72

5) World Investment Report is published by Education and Literacy in the Government of


which of the following organization? India and all States and Union Territories.
a. World Bank  This online junction of different websites and
portals into a single platform will enhance the
b. World Economic Forum
accessibility of information relating to schools
c. International Monetary Fund and will ensure a holistic approach to
transform education sector.
d. UN Conference on Trade and Development
4. c
 India has become the first country in the world
Answers to issue Biometric Seafarer Identity
1. c Document (BSID), capturing the facial bio-
metric data of seafarers.
 India has become the first country in the world
to issue Biometric Seafarer Identity  The new card is in confirmation of
Document (BSID), capturing the facial bio- the Convention No. 185 of the
metric data of seafarers. International Labour Organisation on
BSID.
 The new document will give a foolproof
identification to our seafarers which will  India ratified the Convention in October
facilitate their movement, provide ease of 2015.
getting jobs and help in identifying them from 5. d
any location in the world.
 The BSID introduces modern security
features. It will have a biometric chip 30-08-2019
embedded in it.
1) Consider the following statements with
 A record of each SID issued will be maintained respect to “Fit India Movement”
in a national database and its related
1. It aims to encourage people to inculcate
information will be internationally accessible.
physical activity and sports in their
 In India the BSID project has been taken up in everyday lives.
collaboration with Centre for Development of 2. It was launched on the event of World
Advanced Computing (CDAC), Mumbai. Health Day, 2019.
2. b Which of the statement(s) given above is/are
 The Status of Policing in India Report, correct?
2019 on police adequacy and working a. 1 only
conditions was prepared by the NGO
Common Cause and Lokniti programme of b. 2 only
the Centre for the Study of Developing c. Both 1 and 2
Societies.
d. Neither 1 nor 2
 According to the report, one in two police
personnel surveyed feel that Muslims are likely
to be ―naturally prone‖ to committing crimes. 2) Consider the following statements with
3. c respect to Indian Star Tortoise

 School Education Shagun is an over- 1. It was found in the dry and arid forests
arching initiative to improve school education of both Indian and Sri Lanka.
system by creating a junction for all online 2. It has been listed as vulnerable under
portals and websites relating to various the IUCN Red list.
activities of the Department of School

www.shankariasacademy.com | www.iasparliament.com
73

3. It has been recently listed under the 5) Kodavas are a small martial community
Appendix I of CITES and will now enjoy inhabited in which following states?
the highest degree of protection.
a. Goa
Which of the statement(s) given above is/are
b. Kerala
correct?
c. Karnataka
a. 3 only
d. Tamil Nadu
b. 1 and 2 only
c. 2 and 3 only
6) Coprolites sometimes seen in the news
d. 1, 2 and 3
recently, refers to?
a. Extinct culture
3) Consider the following statements with
b. Fossilised faeces
respect to PAiSA Portal
c. A Period in Iron age
1. It is a centralized IT platform which
simplifies and streamlines release of d. A hard variety of coal
interest subvention under the Rashtriya
Ucchatar Shiksha Abhiyan (RUSA).
2. The web platform has been designed 7) Consider the following statements with
and developed by Ministry of Human respect to Himmat Plus app
Resources and Development (MHRD). 1. It aims to ensure the safety of amaranth
Which of the statement(s) given above is/are pilgrims in the Himalayan region.
correct? 2. It was launched by Ministry of Home
a. 1 only affairs.

b. 2 only Which of the statement(s) given above is/are


correct?
c. Both 1 and 2
a. 1 only
d. Neither 1 nor 2
b. 2 only
c. Both 1 and 2
4) Consider the following statements with
respect to Hurricanes d. Neither 1 nor 2

1. A hurricane is a type of storm called a


tropical cyclone, which forms over 8) Saffir–Simpson scale is associated with
tropical or subtropical waters. which of the following?
2. To qualify as a hurricane, a storm must a. Hurricane
have sustained winds of 74 kmph or
more. b. Earthquake

Which of the statement(s) given above is/are c. Heat waves


correct? d. Volcanic eruption
a. 1 only
b. 2 only 9) Gundla Brahmeswaram Wildlife
c. Both 1 and 2 Sanctuary is located in which of the following
states?
d. Neither 1 nor 2
a. Odisha
b. Karnataka

www.shankariasacademy.com | www.iasparliament.com
74

c. Andhra Pradesh  A tropical cyclone is a rotating low-pressure


weather system that has organized
d. Himachal Pradesh
thunderstorms but no fronts (a boundary
separating two air masses of different
densities).
10) Consider the following pairs
Places in News – Country  Tropical cyclones with maximum sustained
surface winds of less than 39 miles per
1. East Kalimantan – Indonesia hour (mph) are called tropical depressions.
2. Puerto Rico – United States of America  Those with maximum sustained winds of 39
Which of the pair(s) given above is/are mph or higher are called tropical storms.
correctly matched?  When a storm's maximum sustained winds
a. 1 only reach 74 mph, it is called a hurricane.

b. 2 only  The Saffir-Simpson Hurricane Wind Scale is a


1 to 5 rating, or category, based on a
c. Both 1 and 2 hurricane's maximum sustained winds. The
d. Neither 1 nor 2 higher the category, the greater the hurricane's
potential for property damage.
 Hurricane Dorian forecast to reach Florida as
Answers a Category 4 storm.
1. a 5. c
 Indian Prime Minister has recently launched a  The special privilege to own firearms without a
nation-wide Fit India Movement from New license was granted to the Kodavas, a small
Delhi on the occasion of National Sports martial community in Kodagu (Coorg)
Day celebrated on August 29, 2019. District in Karnataka, by the British in
recognition of their martial traditions.
 It is aimed at encouraging people to inculcate
physical activity and sports in their day to day  Kodavas, an ethno-linguistic tribe, consider
life. themselves the original inhabitants of Kodagu
district and are known for their martial
2. d
culture.
3. d
 The community have its rituals around guns
 Deendayan Antyodaya Yojana – and weapons like swords.
National Urban Livelihoods Mission
(DAY-NULM), a flagship mission under  The Kodava community’s special privilege to
the Ministry of Housing and Urban own firearms without a license has come
Affairs has been conferred the prestigious under the spotlight after a writ petition was
SKOCH Governance Gold Award for filed in the Karnataka High Court questioning
its Portal for Affordable Credit and the exemption granted to the community
Interest Subvention Access (PAiSA). under the provisions of the Arms Act, 1959.
6. b
 Launched in November 2018, PAiSA is a
centralized IT platform which simplifies and  Coprolites are fossilised faeces belonging
streamlines release of interest subvention to animals that lived millions of years ago.
under the Mission.
7. d
 It offers end to end online solution for
processing, payment, monitoring and tracking  Delhi Lieutenant Governor has recently
of interest subvention claims from banks on a launched the “QR Code” scheme, which will
monthly basis. be delivered through Delhi Police‘s “Himmat
Plus” app meant to ensure the safety of
4. a women.

www.shankariasacademy.com | www.iasparliament.com
75

 The QR codes will be put on public transport 2) Consider the following statements with
vehicles such as auto-rickshaws and non-app respect to Mount Kun
(black-and-yellow) cabs; passengers, especially
1. It is the second highest peak in the
women, will be able to verify the details of
Indian side of Himalayas.
drivers, and send a distress signal in an
emergency. 2. It is located in Karakoram Range in
Ladakh.
 The Himmat Plus app was launched
by Delhi Police in 2015 to allow users to Which of the statement(s) given above is/are
make a distress call or send an emergency correct?
message to Delhi Police and the emergency a. 1 only
contact.
b. 2 only
8. a
c. Both 1 and 2
 The Saffir–Simpson hurricane wind
scale (SSHWS) classifies hurricanes that d. Neither 1 nor 2
exceed the intensities of tropical depressions
and tropical storms – into five
categories distinguished by the intensities of 3) Consider the following statements with
their sustained winds. respect to Ek Bharat, Vijayi Bharat
9. c 1. It is a year-long nation-wide contact
programme to spread the messages of
10. c Mahatma Gandhi.
 Indonesia‘s President Joko Widodo had 2. It coincides with the 150th foundation
recently announced that the capital of the year of Mahatma Gandhi memorial in
country, which is Jakarta at present, will be Kanyakumari, Tamil Nadu.
relocated to the province of East
Kalimantan on the lesser populated island Which of the statement(s) given above is/are
of Borneo. correct?
a. 1 only

31-08-2019 b. 2 only

1) Consider the following statements with c. Both 1 and 2


respect to State of World Population Report d. Neither 1 nor 2
2019
1. It is the flagship report of the United
Nations Population Fund (UNFPA). 4) Consider the following statements with
respect to Vivekananda rock memorial
2. The report includes, for the first time,
data on women‘s ability to make 1. It is located at the Tri-junction of Indian
decisions over three key areas – sexual Ocean, Bay of Bengal and Arabian Sea in
intercourse with their partner, Kanyakumari.
contraception use and health care. 2. It was built in the year 1970, to
Which of the statement(s) given above commemorate the visit of Swami
is/are incorrect? Vivekananda to Shripada Parai in 1892
for deep meditation and enlightenment.
a. 1 only
Which of the statement(s) given above is/are
b. 2 only correct?
c. Both 1 and 2 a. 1 only
d. Neither 1 nor 2 b. 2 only
c. Both 1 and 2

www.shankariasacademy.com | www.iasparliament.com
76

d. Neither 1 nor 2 2. It promotes sustainable management


and conservation of tropical forests.
Which of the statement(s) given above is/are
5) The e-course on Vulnerability Atlas was
correct?
recently launched by which of the following
ministries? a. 1 only
a. NITI Aayog b. 2 only
b. Ministry of Earth Sciences c. Both 1 and 2
c. Ministry of Science and Technology d. Neither 1 nor 2
d. Ministry of Housing and Urban Affairs
9) Consider the following pairs
6) Consider the following pairs with respect 1. Kunduz – Afghanistan
to GI Tag
2. N'Djamena – Chad
1. Dindigul lock – Telangana
Which of the statement(s) given above is/are
2. Kandangi saree – Tamil Nadu correctly matched?
Which of the pair(s) given above a. 1 only
is/are incorrectly matched?
b. 2 only
a. 1 only
c. Both 1 and 2
b. 2 only
d. Neither 1 nor 2
c. Both 1 and 2
d. Neither 1 nor 2
10) With the reference to the Ancient Indian
history, the term ‗cire perdue‘ process was
used for making of the?
7) Consider the following statements with
respect to Damaram a. Cave Paintings
1. It is a set of drums, made up of wood, b. Metal sculptures
conical in shape, resembling flower
c. Coin mintings
pots.
d. Branding of Horses
2. It belongs to the category of Tata
Vadyas.
Which of the statement(s) given above is/are Answers
correct?
1. d
a. 1 only
 India accounts for over one-sixth of the world‘s
b. 2 only population in 2019 (1.37 billion out of 7.71
c. Both 1 and 2 billion) and has grown at an rate (1.2% per
year between 2010 and 2019) that is just over
d. Neither 1 nor 2 the world growth rate (1.2%), according
to State of the World Population 2019,
the flagship report of the United Nations
8) Consider the following statements with Population Fund (UNFPA).
respect to International Tropical Timber
Organization (ITTO)  The report includes, for the first time, data on
women‘s ability to make decisions over three
1. It is one of the specialized agencies of key areas – sexual intercourse with their
the United Nations (UN). partner, contraception use and health care.

www.shankariasacademy.com | www.iasparliament.com
77

2. d Geographical Indication (GI) tag by The


Geographical Indications Registry in Chennai.
 Mt. Kun (7077M), the second highest peak
of the Nun Kun massif is located in the 7. a
upper Suru Valley.
 Damaram, made of wood, is a set of drums,
 Being one of the highest peaks in the Zanskar conical in shape, resembling flower pots.
Range and on the Indian side of the Line of
 It belongs to the category of Avanaddha
Control, the twin peaks of Nun (7135M) and
Vadyas.
Kun are separated by a 4km long snow
plateau.  One face of the instrument is covered with cow
3. d hide while the other, with that of the goat.

 Kanyakumari-based Vivekanand Kendra and  Two sticks, made from Arali kuchi, are used
its state units will launch a year-long nation- for playing this instrument. One stick is
wide contact programme to spread the straight while the other is a bit curved.
messages of Swami Vivekananda. 8. b
 The mass contact programme, titled "Ek  The International Tropical Timber
Bharat Vijayi Bharat", coincides with the Organization (ITTO) is
50th foundation year of Vivekanand Rock an intergovernmental
Memorial in Kanyakumari and will be organization promoting the sustainable
launched on September 1, 2019. management and conservation of tropical
4. c forests and the expansion and diversification
of international trade in tropical timber from
 Vivekanand rock memorial has been an iconic sustainably managed and legally harvested
landmark at the Tri-junction of Indian forests.
Ocean, Bay of Bengal and Arabian Sea in
9. c
Kanyakumari.
10. b
 As its name implies, it is essentially a sacred
monument, built by the Vivekananda Rock
Memorial Committee to commemorate the
visit of Swamy Vivekananda to ―Shripada
Parai‖ during 24th, 25th and 26th December
1892 for deep meditation and enlightenment.
5. d
 The Ministry of Housing & Urban
Affairs in collaboration of School of Planning
& Architecture (SPA), New Delhi and Building
Materials & Technology Promotion Council
(BMTPC).
 It is a unique course that offers awareness and
understanding about natural hazards, helps
identify regions with high vulnerability with
respect to various hazards (earthquakes,
cyclones, landslides, floods, etc.) and specifies
district-wise level of damage risks to the
existing housing stock.
6. a
 Two well-known products from Tamil
Nadu — Dindigul lock and Kandangi
saree — have been recently given the

www.shankariasacademy.com | www.iasparliament.com

You might also like